1. A 28-year-old patient complains of sensation of a foreign body under the eyelids, eyelid heaviness, dry eyes, and lacrimation, especially in the evening. Objectively, the edges of the eyelids are hyperemic and edematous, isolated small scales can be observed between the eyelashes, the conjunctiva of the eyelids and eyeball is unchanged, the optical media and fundus are normal. Visual acuity 1.0. What is the most likely diagnosis In this case?
A. Keratitis
B. Blepharitis
C. Stye
D. Spring catarrh
E. Chalazion
Correct Answer: B. Blepharitis
Explanation:
Blepharitis is an inflammation of the eyelid margins, commonly caused by bacterial infections (e.g., Staphylococcus), seborrheic dermatitis, or Demodex mite infestation. The symptoms described—foreign body sensation, eyelid heaviness, dry eyes, lacrimation (especially in the evening), hyperemic and edematous eyelid margins, and small scales between the eyelashes—are characteristic of blepharitis. The absence of conjunctival or corneal involvement supports this diagnosis.
Why Other Options Are Incorrect:
- A. Keratitis:
- Keratitis (corneal inflammation) typically presents with pain, photophobia, and corneal opacity, which are not mentioned in this case. The patient’s visual acuity is normal (1.0), ruling out significant corneal involvement.
- C. Stye (Hordeolum):
- A stye is an acute localized infection of an eyelash follicle or meibomian gland, presenting as a painful, red, swollen lump on the eyelid margin. The patient does not have a localized lump, tenderness, or acute inflammatory signs.
- D. Spring Catarrh (Vernal Keratoconjunctivitis):
- This is a chronic allergic condition seen in young males, often during spring/summer. It presents with intense itching, photophobia, mucoid discharge, and cobblestone papillae on the conjunctiva, none of which are present in this case.
- E. Chalazion:
- A chalazion is a chronic, painless, firm nodule due to blockage of a meibomian gland. Unlike blepharitis, it is not associated with scales or eyelid margin inflammation. The patient’s symptoms do not indicate a localized nodule.
Conclusion:
The patient’s symptoms and clinical findings strongly indicate blepharitis, making option B the most likely diagnosis.
2. A 5-year-old girl developed repeated vomiting and loss of consciousness on the fifth day after the onset of an acute respiratory viral infection. According to the patient’s medical history, her mother was giving her acetylsalicylic acid and metamizole sodium without a prescription. Objectively, the following is observed: the score of 5- 6 at the Glasgow Coma Scale, hemorrhagic syndrome, hepatomegaly, blood pressure 100/60 mm Hg. Blood test results: hypoglycemia, normal bilirubin and creatinine levels, hyperammonemia, a tenfold increase in ALT and AST levels, decreased prothrombin index and albumin levels. What emergency condition has developed in the child?
A. Acute renal failure
B. Ketoacidotic coma
C. Anaphylactic shock
D. Acute liver failure
E. Acute adrenal insufficiency
Correct Answer: D. Acute Liver Failure
Explanation:
The child presents with vomiting, loss of consciousness, hemorrhagic syndrome, hepatomegaly, and severe liver dysfunction (markedly elevated ALT and AST, hypoglycemia, hyperammonemia, decreased prothrombin index and albumin levels). These findings strongly indicate acute liver failure (ALF).
The history of acetylsalicylic acid (aspirin) use in a child with a recent viral infection suggests Reye’s syndrome, a rare but severe condition characterized by acute liver failure and encephalopathy. Reye’s syndrome is associated with aspirin use in viral illnesses, leading to mitochondrial dysfunction and hepatic steatosis.
Why Other Options Are Incorrect:
- A. Acute Renal Failure:
- The child does not have azotemia (elevated creatinine, urea), oliguria/anuria, or electrolyte imbalances suggestive of kidney failure.
- Normal creatinine levels make this diagnosis unlikely.
- B. Ketoacidotic Coma:
- Diabetic ketoacidosis (DKA) presents with polyuria, polydipsia, Kussmaul breathing, fruity odor, and metabolic acidosis with ketonuria.
- This patient has hypoglycemia rather than hyperglycemia, making DKA unlikely.
- C. Anaphylactic Shock:
- Anaphylaxis presents with hypotension, bronchospasm, urticaria, and angioedema, none of which are present in this case.
- The patient has normal blood pressure (100/60 mmHg), ruling out shock.
- E. Acute Adrenal Insufficiency:
- Features of adrenal crisis include hypotension, severe fatigue, hyperkalemia, hyponatremia, and hypoglycemia.
- This patient has hepatic dysfunction with markedly elevated liver enzymes, which is not a feature of adrenal insufficiency.
Conclusion:
The acute liver dysfunction with encephalopathy and aspirin use in a viral illness strongly suggests Reye’s syndrome, leading to acute liver failure (ALF). Thus, option D is the correct answer.
3. A 40-year-old man complains of fever, chiIls, tachycardia, and pain in the area of his li- ver. Ultrasound detects a hypoechoic focus with clear contours and homogeneous contents in the right lobe of the liver, against the background of which there is a high-intensity inclusion. The CT scan revealed a round mass with smooth clear contours and a gas bubble in its upper part. Blood tests detect leukocytosis and increased ESR. What is the most likely diagnosis in this case?
A. Liver abscess
B. Capillary hemangioma of the liver
C. Cavernous hemangioma of the liver
D. Solitary hepatic cyst
E. Cancer metastasis
Correct Answer: A. Liver Abscess
Explanation:
The patient presents with fever, chills, tachycardia, and liver pain, which are characteristic of an infectious process. The ultrasound and CT scan findings—a hypoechoic focus with homogeneous contents and a gas bubble in the upper part—strongly suggest a liver abscess, likely pyogenic.
Pyogenic liver abscesses often result from biliary infections, portal vein infections, hematogenous spread, or direct extension from adjacent infections. The presence of leukocytosis and increased ESR further supports an inflammatory or infectious etiology.
Why Other Options Are Incorrect:
- B. Capillary Hemangioma of the Liver:
- These are small, well-circumscribed vascular tumors and are usually asymptomatic.
- They appear as hyperechoic on ultrasound, not hypoechoic.
- No fever, chills, or gas bubbles are associated with hemangiomas.
- C. Cavernous Hemangioma of the Liver:
- This is the most common benign liver tumor, often found incidentally.
- It appears as a hyperechoic lesion with possible enhancement on CT, but it does not contain gas bubbles or cause fever and chills.
- D. Solitary Hepatic Cyst:
- Simple hepatic cysts are anechoic (fluid-filled), have smooth walls, and do not contain gas bubbles.
- They are usually asymptomatic unless very large, causing mass effect.
- No systemic inflammatory response (fever, leukocytosis, high ESR) is seen in hepatic cysts.
- E. Cancer Metastasis:
- Liver metastases typically appear as multiple lesions with irregular contours and variable echogenicity rather than a single hypoechoic lesion with gas.
- They are usually not associated with fever, chills, or gas formation.
- Gas bubbles in a liver lesion strongly suggest an infectious rather than neoplastic cause.
Conclusion:
The presence of fever, chills, liver pain, leukocytosis, increased ESR, and imaging findings of a hypoechoic lesion with a gas bubble strongly supports the diagnosis of a liver abscess, making option A the correct answer.
- A 28-year-old patient complains of severe shortness of breath during even mild physical exertion, dizziness, forced body position (semi- sitting), and subfebrile body temperature. According to the patient’s medical history, her condition was gradually deteriorating over the course of the last 1.5 months after a case of an acute respiratory viral infection. Auscultation detects harsh respiration and dry crackles, the heart sounds are rhythmic and severely muffled. Blood pressure 110/70 mm Hg. Chest X-ray shows that the shadow of the heart is severely expanded in both directions, the cardiac waistline is smoothed out. What is the most likely diagnosis in this case?
A. Pulmonary artery stenosis
B. Mitral stenosis
C. Exudative pericarditis
D. Coarctation of the aorta
E. Mitral valve insufficiency
Correct Answer: C. Exudative Pericarditis
EExplanation:
Exudative pericarditis, also known as pericardial effusion, is characterized by the accumulation of fluid in the pericardial sac, leading to muffled heart sounds, dyspnea (especially when lying flat), and an enlarged cardiac silhouette on chest X-ray. The key features in this case supporting exudative pericarditis include:
- Gradual worsening over 1.5 months after a viral infection (suggesting viral pericarditis leading to effusion).
- Shortness of breath and dizziness, which occur due to cardiac compression by the fluid.
- Forced semi-sitting position (orthopnea), a classic sign in pericardial effusion.
- Subfebrile temperature, which can be seen in inflammatory conditions.
- Muffled heart sounds, indicating a large effusion dampening heart sounds.
- Chest X-ray findings of an enlarged cardiac silhouette with smoothed cardiac waistline, characteristic of pericardial effusion.
Why Other Options Are Incorrect:
A. Pulmonary artery stenosis
- Pulmonary artery stenosis typically presents with a harsh systolic murmur, differential cyanosis, and right ventricular hypertrophy, none of which are mentioned in this case.
- It does not typically cause muffled heart sounds or cardiac silhouette enlargement on X-ray.
B. Mitral stenosis
- Mitral stenosis is characterized by dyspnea on exertion, orthopnea, and a diastolic murmur with an opening snap—none of which are noted here.
- Chest X-ray in mitral stenosis typically shows left atrial enlargement rather than a globally enlarged heart.
D. Coarctation of the aorta
- Coarctation of the aorta presents with hypertension in the upper extremities and weak pulses in the lower extremities, which are not mentioned here.
- The CXR would show rib notching rather than a globally enlarged heart.
E. Mitral valve insufficiency
- Mitral regurgitation presents with holosystolic murmur at the apex radiating to the axilla, which is absent in this case.
- While mitral regurgitation can cause heart enlargement, it usually leads to left atrial and left ventricular enlargement, rather than the globally enlarged heart seen in this case.
Conclusion:
The patient’s gradual deterioration after a viral infection, muffled heart sounds, dyspnea, subfebrile temperature, and enlarged cardiac silhouette on CXR strongly suggest exudative pericarditis as the most likely diagnosis.
- A 56-year-old overweight patient complains of pain in her knee joints. The pain intensifies when she walks, climbs stairs, or remains standing for a long time. The disease onset was 5 years ago. Objectively, the knee joints are deformed, edematous, and painful during movements. X-ray of the knee joints shows joint space narrowing, subchondral sclerosis, and marginal osteophytes. What is the most likely diagnosis in this case?
A. Osteoarthrosis
B. Reactive arthritis
C. Rheumatoid arthritis.
D. Rheumatic arthritis
E. Gout
Correct Answer: A. Osteoarthrosis (Osteoarthritis)
Explanation:
Osteoarthrosis (osteoarthritis) is a chronic degenerative joint disease that primarily affects weight-bearing joints such as the knees, hips, and spine. It is caused by progressive cartilage degeneration and is associated with aging, obesity, and mechanical stress.
Key Features Supporting the Diagnosis:
- Patient Demographics: 56-year-old, overweight (major risk factor).
- Symptoms:
- Chronic knee pain that worsens with activity (e.g., walking, climbing stairs, prolonged standing).
- Gradual onset over 5 years (consistent with a degenerative process).
- Joint deformity, edema, and pain with movement, but no significant redness or warmth (inflammatory arthritis would present with these).
- X-ray Findings (Classic for Osteoarthritis):
- Joint space narrowing (due to cartilage loss).
- Subchondral sclerosis (increased bone density beneath the cartilage).
- Marginal osteophytes (bone spurs at joint margins, a hallmark of osteoarthritis).
Why Other Options Are Incorrect:
B. Reactive Arthritis
- Typically follows a recent infection (e.g., gastrointestinal or urogenital infection), which is not mentioned here.
- Presents as asymmetric arthritis of the lower extremities, often with conjunctivitis and urethritis (Reiter’s syndrome).
- X-ray would not show osteophytes or joint space narrowing.
C. Rheumatoid Arthritis (RA)
- Typically affects small joints (hands, wrists, feet) in a symmetrical pattern, rather than isolated knee involvement.
- Symptoms include morning stiffness lasting >1 hour, which is not described here.
- X-ray in RA shows joint erosions and periarticular osteopenia, not osteophytes and subchondral sclerosis.
D. Rheumatic Arthritis (Rheumatic Fever)
- Usually follows a streptococcal throat infection, with migratory polyarthritis affecting large joints transiently.
- Does not cause chronic progressive joint degeneration.
- No joint space narrowing or osteophytes on imaging.
E. Gout
- Presents with acute, severe pain and swelling, typically affecting the first metatarsophalangeal (MTP) joint (big toe) rather than the knees.
- X-ray findings in chronic gout include “punched-out” erosions with overhanging edges (“rat-bite lesions”), not osteophytes or subchondral sclerosis.
- Uric acid levels and tophi (chalky deposits) are often present in advanced cases.
Conclusion:
The gradual onset, weight-bearing joint involvement, worsening pain with activity, and characteristic X-ray findings (joint space narrowing, osteophytes, and subchondral sclerosis) strongly indicate Osteoarthrosis (Osteoarthritis), making option A the correct answer.
- A patient, who has hemorrhages in the form of dark ark red spots on the skin, underwent the tongue test with Tillman’s reagent on an empty stomach, one hour before a meal discoloration occurred in 58 seconds. What vitamin is deficient in this case, as indicated by the test?
A. Vitamin A
B. Vitamin pp
C. Vitamin D3
D. Vitamin C
E. Vitamin E
Correct Answer: D. Vitamin C
Explanation:
The patient presents with dark red hemorrhages on the skin, which is a classic symptom of scurvy, a disease caused by vitamin C (ascorbic acid) deficiency. Vitamin C is crucial for collagen synthesis, and its deficiency leads to weakened capillaries, resulting in easy bruising, petechiae, and hemorrhages.
The Tillman’s reagent test (or “decolorization test”) is used to assess vitamin C levels. It involves the use of Tillman’s reagent (2,6-dichlorophenol-indophenol, DCPIP), which changes color in the presence of vitamin C. A rapid decolorization time (less than 60 seconds) suggests severe vitamin C deficiency.
Why Other Options Are Incorrect:
- A. Vitamin A – Deficiency in vitamin A leads to night blindness (nyctalopia), xerophthalmia (dry eyes), and keratomalacia but does not cause hemorrhages or affect capillary integrity.
- B. Vitamin PP (Niacin, B3) – Niacin deficiency causes pellagra, characterized by the 3 D’s: dermatitis, diarrhea, and dementia, but it does not lead to skin hemorrhages.
- C. Vitamin D3 – Deficiency leads to rickets (in children) and osteomalacia (in adults), causing bone deformities and muscle weakness, but not hemorrhages.
- E. Vitamin E – While vitamin E is an antioxidant that protects cell membranes, its deficiency mainly causes neuromuscular disorders and hemolytic anemia, not hemorrhagic manifestations.
Thus, the correct answer is Vitamin C deficiency, which leads to scurvy and easy bruising as seen in this patient.
- A 32-year-old patient looks at the pattern on the wallpaper and sees the lines begin to move and form silhouettes of fantastic animals. Instead of a lighting fixture hanging from the ceiling, he sees a giant octopus. What psychopathological symptom is it?
A. Pseudohallucinations
B. Visual hallucinations
C. Derealization
D. Functional hallucinations
E. Pareidolic illusions
Correct Answer: E. Pareidolic illusions
Explanation:
Pareidolic illusions occur when a person misinterprets real objects or patterns, seeing them as meaningful images or figures that do not actually exist. In this case, the patient sees silhouettes of animals in wallpaper patterns and a giant octopus instead of a lighting fixture, which are characteristic of pareidolia. This phenomenon occurs when the brain imposes familiar shapes onto random patterns.
Why Other Options Are Incorrect:
- A. Pseudohallucinations – These are hallucinations that the patient recognizes as not real (e.g., hearing voices but knowing they are imaginary). In contrast, pareidolic illusions arise from real external stimuli.
- B. Visual hallucinations – True visual hallucinations occur without any external stimuli. The patient in this case is misinterpreting actual objects, which is an illusion, not a hallucination.
- C. Derealization – This is a feeling that the world around the patient is unreal, dreamlike, or distorted, but it does not involve seeing specific images or figures.
- D. Functional hallucinations – These occur only in the presence of a real stimulus (e.g., hearing voices only when water is running), but pareidolic illusions are not true hallucinations.
Thus, the correct answer is pareidolic illusions, as the patient is misinterpreting real patterns as recognizable images.
8.A 30-year-old woman complains of fever of 38.7°C, pain in the lower abdomen, and dysuric disorders. She has history of an artificial abortion performed 3 days ago. Bimanual examination detects that the cervix is cylindrical, the opening is closed, the body of the uterus is slightly enlarged, painful, and soft. The uteri- ne appendages are not palpable. Purulent- hemorrhagic discharge is being produced. Blood test results: leukocytes10109/L, band neutrophils 12%. What is the most likely diagnosis in this case?
A. Relvioperitonitis
B .Endometriosis
C. Acute cystitis
D. Acute salpingo-oophoritis
E. Acute endometrities
Correct Answer: E. Acute Endometritis
Explanation:
This 30-year-old woman presents with fever (38.7°C), lower abdominal pain, dysuria, and purulent-hemorrhagic discharge, which developed three days after an artificial abortion. On examination, the uterus is slightly enlarged, painful, and soft, while the cervix remains closed. Her blood test shows leukocytosis with a left shift (12% band neutrophils), indicating an acute infection.
These findings are highly suggestive of acute endometritis, an infection of the endometrial lining commonly caused by ascending bacterial infections following procedures such as abortion, delivery, or intrauterine device (IUD) placement.
Why Other Options Are Incorrect:
- A. Pelvioperitonitis (Relvioperitonitis) – This is an advanced complication of pelvic infections, involving inflammation of the peritoneum. It presents with severe abdominal pain, rebound tenderness, guarding, and paralytic ileus, which are absent in this case.
- B. Endometriosis – This condition is characterized by chronic pelvic pain, dysmenorrhea, and infertility, rather than an acute febrile illness. The absence of cyclic pain or endometriotic nodules makes this diagnosis unlikely.
- C. Acute Cystitis – Although the patient has dysuria, acute cystitis primarily presents with frequent urination, urgency, suprapubic pain, and no fever or uterine tenderness. The presence of uterine pain and purulent discharge suggests a uterine infection rather than a bladder infection.
- D. Acute Salpingo-Oophoritis (Pelvic Inflammatory Disease, PID) – This condition involves infection of the fallopian tubes and ovaries, often presenting with bilateral adnexal tenderness and possibly a palpable mass if an abscess forms. In this case, the uterine appendages are not palpable, making acute endometritis the more likely diagnosis.
Thus, the correct answer is acute endometritis, a common post-abortion infection. Prompt antibiotic therapy is essential to prevent complications like pelvic inflammatory disease (PID) or septicemia.
9.A 28-year-old woman at 36 weeks of her pregnancy complains of headache, nausea, vomiting, and pain in the right hypochondrium. Objectively, the following is observed: skin pallor, icteric mucosa, the blood pressure of 160/100 mm Hg on both hands, edema of the legs and anterior abdominal wall, hemorrhages at the injection sites. Palpation detects sharp. pain in the epigastric region and an enlarged li- ver. Blood test results: elevated ALT and AST levels, thrombocytopenia, hemoglobin 85 g/L, hyperbilirubinemia. What complication has developed in the pregnant woman?
A. Amniotic fluid embolism
B. Disseminated intravascular coagulation syndrome
Ć. Eclampsia
D. Liver rupture
E. HELLP syndrome
Correct Answer: E. HELLP Syndrome
Explanation:
This 28-year-old woman at 36 weeks of pregnancy presents with hypertension (160/100 mmHg), headache, nausea, vomiting, right upper quadrant (RUQ) pain, pallor, jaundice, edema, hemorrhages at injection sites, and laboratory findings of thrombocytopenia, elevated liver enzymes (ALT, AST), anemia (Hb 85 g/L), and hyperbilirubinemia.
These findings are highly characteristic of HELLP syndrome, which stands for:
- Hemolysis (anemia, hyperbilirubinemia)
- Elevated Liver enzymes (ALT, AST)
- Low Platelets (thrombocytopenia)
HELLP syndrome is a severe complication of preeclampsia, usually occurring in the third trimester, and presents with hypertension, liver dysfunction, hemolysis, and coagulation abnormalities. The RUQ pain is due to hepatic involvement and capsular distension of the liver.
Why Other Options Are Incorrect:
- A. Amniotic Fluid Embolism – This presents acutely with sudden cardiopulmonary collapse, respiratory distress, and disseminated intravascular coagulation (DIC). The gradual onset of symptoms in this case makes AFE unlikely.
- B. Disseminated Intravascular Coagulation (DIC) Syndrome – DIC can occur secondary to HELLP syndrome, but it is not the primary diagnosis here. DIC presents with widespread bleeding, shock, and multiorgan failure, which are not the dominant findings in this case.
- C. Eclampsia – Eclampsia is characterized by seizures in a patient with preeclampsia. This patient has hypertension and organ dysfunction but no seizures, making eclampsia unlikely.
- D. Liver Rupture – Liver rupture is a rare but life-threatening complication of severe HELLP syndrome. However, the absence of signs of hemodynamic shock (severe hypotension, acute abdomen, peritoneal signs) makes rupture less likely at this stage.
Thus, the correct answer is HELLP syndrome, a severe form of preeclampsia that requires urgent obstetric intervention, antihypertensive therapy, corticosteroids (for fetal lung maturity), and delivery as definitive management.
10 A 27-year-old patient complains of regular menstruation with delays of 2-3 months, significant body weight gain, hirsutism. She has been married for the last 5 years and had no pregnancies. Vaginal examination shows that the uterus is under 4 cm in size, on both sides there are dense mobile ovaries 4-5 cm in size that are painless during palpation. Ultrasound detects bilateral enlargement of the ovaries to 10.5 cm³, thickening of the ovarian capsule, and 8 immature follicles 5-6 mm in diameter, located on the periphery like a “necklace”. What is the most likely diagnosis in this case?
A. Adnexal tuberculosis
B. Hypomenstrual syndrome
C. Bilateral ovarian cysts
D.Sclerocystic Ovary syndrome
E. Chronic bilateral salpingitise
Correct Answer: D. Sclerocystic Ovary Syndrome (Polycystic Ovary Syndrome – PCOS)
Explanation:
This 27-year-old patient presents with:
- Oligomenorrhea (irregular menstruation with delays of 2-3 months)
- Weight gain
- Hirsutism (excessive hair growth, suggesting hyperandrogenism)
- Infertility (married for 5 years with no pregnancies)
- Bilateral ovarian enlargement (10.5 cm³) with thickened capsules
- Multiple small peripheral cysts (8 immature follicles, 5-6 mm, “necklace” pattern on ultrasound)
These findings are hallmark features of Polycystic Ovary Syndrome (PCOS), also called Sclerocystic Ovary Syndrome (SCOS). PCOS is an endocrine disorder characterized by chronic anovulation, hyperandrogenism, and polycystic ovarian morphology. The condition is commonly associated with insulin resistance, obesity, and metabolic syndrome.
Why Other Options Are Incorrect:
- A. Adnexal Tuberculosis – Typically presents with chronic pelvic pain, fever, weight loss, and caseous granulomas on imaging. The patient lacks systemic signs of tuberculosis.
- B. Hypomenstrual Syndrome – Refers to light or infrequent menstruation but does not cause ovarian enlargement, multiple follicles, or hyperandrogenism.
- C. Bilateral Ovarian Cysts – Simple ovarian cysts are usually larger (>3 cm) and do not cause hormonal symptoms like hirsutism or menstrual irregularities. PCOS, on the other hand, presents with multiple small follicles rather than a few large cysts.
- E. Chronic Bilateral Salpingitis – This is a fallopian tube infection, presenting with pelvic pain, fever, and tubal thickening, rather than ovarian enlargement with a “necklace” appearance on ultrasound.
Thus, the correct answer is Sclerocystic Ovary Syndrome (PCOS). Management includes lifestyle modifications, hormonal therapy (oral contraceptives), and fertility treatment if pregnancy is desired.
11. A 37-year-old patient complains of throbbing headache in the frontal region, frequent episodes of nausea, palpitations, and elevated blood pressure of 240/140 mm Hg. According to the patient’s medical history, two days ago she experienced an episode of debilitating headache, palpitations, pulsation of blood vessels, dyspnea, and fear of approaching death. At that time, her face became pale and her hands and feet became sweaty. After examination, elevated levels of metanephrines were detected in patient’s urine. What is the most likely diagnosis in this case?
A. Cushing disease
B. Cushing syndrome
C. Essential hypertension
D. Primary hyperaldosteronism
E Pheochromocytoma
Correct Answer: E. Pheochromocytoma
Explanation:
Pheochromocytoma is a catecholamine-secreting tumor, usually located in the adrenal medulla. It leads to episodic or sustained hypertension along with symptoms such as:
- Severe throbbing headaches
- Palpitations
- Profuse sweating (diaphoresis)
- Pallor
- Paroxysmal hypertension (as seen in this patient with a BP of 240/140 mmHg)
- Anxiety or a sense of impending doom
The elevated urinary metanephrines confirm the presence of excessive catecholamine production, making pheochromocytoma the most likely diagnosis.
Why Other Options Are Incorrect:
A. Cushing disease – Incorrect because:
- Cushing disease (caused by an ACTH-secreting pituitary adenoma) leads to chronic rather than episodic hypertension.
- Symptoms include weight gain, central obesity, moon facies, striae, and proximal muscle weakness, none of which are described here.
B. Cushing syndrome – Incorrect because:
- Cushing syndrome refers to any cause of excess cortisol (not just pituitary adenomas).
- It typically presents with hyperglycemia, osteoporosis, truncal obesity, facial plethora, and easy bruising, none of which are present in this case.
C. Essential hypertension – Incorrect because:
- Essential hypertension develops gradually and does not cause the episodic hypertensive crises seen in this patient.
- It is also not associated with elevated urinary metanephrines.
D. Primary hyperaldosteronism – Incorrect because:
- This condition (Conn’s syndrome) leads to hypertension with hypokalemia, causing muscle weakness and polyuria, which are not mentioned here.
- Unlike pheochromocytoma, it does not cause episodic headaches, sweating, or palpitations.
Thus, pheochromocytoma is the best answer given the symptoms and biochemical findings.
- What type of hospital is specifically organized to provide humane, customized, and family-oriented facilities for the care of dying patients? A. Hospice
B. Day-case care
C. Multidisciplinary hospital
D. Specialized hospital (dispensary)
E. Sanatorium
Correct Answer: A. Hospice
Explanation:
A hospice is a healthcare facility specifically designed to provide palliative and end-of-life care for terminally ill patients. The primary goal is to offer comfort, dignity, and emotional support rather than curative treatment. Hospices focus on pain management, symptom relief, and psychological support for both the patient and their family. Care is usually customized, humane, and family-centered, making it the most appropriate choice in this scenario.
Why Other Options Are Incorrect:
B. Day-case care – Incorrect because:
- Day-case care refers to medical procedures or treatments where patients are admitted and discharged on the same day (e.g., minor surgeries, chemotherapy).
- It is not designed for long-term or end-of-life care.
C. Multidisciplinary hospital – Incorrect because:
- These hospitals provide a wide range of specialized medical services but are not specifically focused on palliative care.
- They may treat terminally ill patients, but they do not emphasize customized, family-centered care for the dying.
D. Specialized hospital (dispensary) – Incorrect because:
- A specialized hospital focuses on treating specific diseases or medical conditions (e.g., cancer hospitals, psychiatric hospitals).
- It is not specifically dedicated to end-of-life care.
E. Sanatorium – Incorrect because:
- Sanatoriums are long-term care facilities traditionally used for chronic diseases like tuberculosis or pulmonary conditions.
- They focus on rehabilitation and recovery, not palliative or end-of-life care.
Thus, hospice is the best answer for a facility dedicated to compassionate care for dying patients.
- A 5-year-old boy has been hospitalized unconscious. According to his is parents, the child has a 2-year-long history of severe decompensated type 1 diabetes mellitus. Objectively, the skin is dry, its turgor is reduced, there is a smell of acetone from the oral cavity, Kussmaul breathing is observed. Blood pressure 100/60 mm Hg, heart rate 100/min. Blood glucose 16.4 mmol/L. What type of coma has developed in the child?
A. Hyperosmolar coma
B. Hypoglycemic coma
C. Cerebral coma
D. Ketoacidotic coma
E. Lactic acid coma
Correct Answer: D. Ketoacidotic coma
Explanation:
The child has a history of type 1 diabetes mellitus with severe decompensation and presents with:
- Dry skin and reduced turgor → Suggests dehydration
- Acetone smell from the mouth → Indicates ketone production
- Kussmaul breathing → Deep, labored breathing that compensates for metabolic acidosis
- Hyperglycemia (16.4 mmol/L) → Elevated blood sugar levels
These findings strongly suggest diabetic ketoacidosis (DKA), which can progress to ketoacidotic coma if untreated.
DKA occurs due to insulin deficiency, leading to excessive lipolysis, ketone accumulation, and metabolic acidosis. This leads to dehydration, electrolyte imbalance, and eventually coma if not corrected.
Why Other Options Are Incorrect:
A. Hyperosmolar coma – Incorrect because:
- Occurs in type 2 diabetes mellitus and is associated with extremely high blood glucose levels (>33 mmol/L), which is not the case here (16.4 mmol/L).
- It typically lacks significant ketone production and does not present with acetone breath or Kussmaul breathing.
B. Hypoglycemic coma – Incorrect because:
- Hypoglycemic coma is due to low blood glucose (<3.0 mmol/L).
- The patient has high blood glucose (16.4 mmol/L), ruling out hypoglycemia.
- Symptoms of hypoglycemia include tremors, sweating, confusion, and seizures, not acetone breath or Kussmaul breathing.
C. Cerebral coma – Incorrect because:
- This is a broad, nonspecific term for coma caused by severe brain injury, stroke, or trauma.
- It does not explain the metabolic findings (hyperglycemia, acidosis, Kussmaul breathing).
E. Lactic acid coma – Incorrect because:
- Lactic acidosis occurs due to shock, hypoxia, or sepsis, leading to excessive lactate production.
- It does not cause acetone breath or Kussmaul breathing.
- Blood lactate levels, not glucose or ketones, would be elevated.
Thus, ketoacidotic coma is the best answer, as all the symptoms align with diabetic ketoacidosis (DKA), a life-threatening complication of type 1 diabetes.
14. A 5-year-old girl has been hospitalized wi- th an electrical injury. Objectively, the child’s condition is extremely severe, she is unconscious and incapable of unassisted breathing. During cardiopulmonary resuscitation, ECG shows waves of varying shape and amplitude with the rate of 320/min. There is no pulse on the peripheral vessels and central arteries. What first aid must be provided in this case?
A. Administration of a lidocaine solution 20 mg intramuscularly
B-Electric defibrillation
C. Open cardiac massage
D. Transfusion of crystalloids 10 mg/kg intravenously
E. Tracheal intubation
Correct Answer: B. Electric defibrillation
Explanation:
The child has suffered an electrical injury and is unconscious, not breathing, and pulseless. The ECG shows waves of varying shape and amplitude at 320/min, which is characteristic of ventricular fibrillation (VF) or pulseless ventricular tachycardia (pVT)—both are life-threatening arrhythmias that require immediate defibrillation as the first-line treatment.
Defibrillation is crucial to restore a normal heart rhythm. Delaying defibrillation reduces survival chances significantly.
Why Other Options Are Incorrect:
A. Administration of a lidocaine solution 20 mg intramuscularly – Incorrect because:
- Lidocaine is an antiarrhythmic drug that may be used for ventricular arrhythmias, but in VF/pVT, immediate defibrillation is the priority.
- IM injection is not effective in this emergency setting.
C. Open cardiac massage – Incorrect because:
- Open cardiac massage is only performed in cases of severe chest trauma or cardiac surgery when closed chest compressions are ineffective.
- In VF/pVT, defibrillation is the key intervention before considering advanced procedures.
D. Transfusion of crystalloids 10 mg/kg intravenously – Incorrect because:
- IV fluids are useful in shock or hypovolemia, but they do not treat VF/pVT.
- The priority is restoring cardiac rhythm through defibrillation.
E. Tracheal intubation – Incorrect because:
- Airway management is important, but defibrillation takes priority.
- If VF/pVT persists, intubation may be needed for oxygenation after defibrillation and CPR.
Thus, electric defibrillation is the most critical and immediate life-saving intervention in this case.
(15) A 25-year-old patient complains of general weakness, low appetite, heaviness and pain in the right hypochondrium, nausea, and dark urine. According to the patient’s medical hi- story, the patient was undergoing outpatient treatment for an acute respiratory viral infection for 3 days. Objectively, icteric sclerae, vesicular respiration, and rhythmic heart sounds are observed. The abdomen is soft and painful in the right hypochondrium. Hepatomegaly is observed. What is the most likely diagnosis in this case?
A. Leptospirosis
B. Viral hepatitis A
C. Pscudotuberculosis
D. Calculous cholecystitis
E. Influenza
Correct Answer: B. Viral hepatitis A
Explanation:
The patient presents with:
- General weakness, low appetite, nausea → Common symptoms of viral infections.
- Dark urine (choluria) → Indicates bilirubinuria, a hallmark of hepatic dysfunction.
- Icteric sclerae (jaundice) → Suggests hyperbilirubinemia, commonly seen in hepatitis.
- Pain and hepatomegaly in the right hypochondrium → Consistent with liver inflammation.
- Recent history of a viral illness → Viral hepatitis A (HAV) is often preceded by flu-like symptoms before jaundice appears.
HAV is a self-limiting disease that spreads via the fecal-oral route and commonly occurs in young adults. It is characterized by acute onset jaundice, dark urine, hepatomegaly, and elevated liver enzymes.
Why Other Options Are Incorrect:
A. Leptospirosis – Incorrect because:
- Leptospirosis can cause jaundice and liver involvement, but it typically presents with severe myalgia (especially in the calves), conjunctival suffusion, and renal dysfunction (acute kidney injury), which are not mentioned in this case.
- Leptospirosis is associated with exposure to contaminated water or animals, which is not indicated here.
C. Pseudotuberculosis – Incorrect because:
- This is a bacterial infection caused by Yersinia pseudotuberculosis, which mimics appendicitis or scarlet fever rather than viral hepatitis.
- It is not associated with jaundice or hepatomegaly but presents with fever, rash, and lymphadenopathy.
D. Calculous cholecystitis – Incorrect because:
- Cholecystitis causes right upper quadrant pain and tenderness, but it is typically colicky and severe rather than a generalized discomfort.
- There is no mention of gallstones, fever, or Murphy’s sign (pain on inspiration when pressing the gallbladder area).
- It does not cause dark urine or jaundice unless there is a bile duct obstruction.
E. Influenza – Incorrect because:
- Influenza causes fever, myalgia, sore throat, and respiratory symptoms, but it does not typically lead to jaundice, dark urine, or hepatomegaly.
- The patient initially had viral symptoms but later developed hepatic signs, which points toward viral hepatitis rather than a respiratory virus.
Thus, viral hepatitis A is the most likely diagnosis, given the combination of jaundice, hepatomegaly, dark urine, and a prodromal viral illness
16. A 28-year-old patient with acute gastrointestinal bleeding of the III degree was transfused with 1000 mL of preserved packed erythrocytes of the same group. What must be used during the hemotransfusion to prevent citrate intoxication?
A. Sodium chloride
B. Calcium chloride
C. Lithium chloride
D. Potassium gluconate
E. Magnesium sulfate
Correct Answer: B. Calcium chloride
Explanation:
During massive blood transfusion, particularly with preserved packed erythrocytes, there is a risk of citrate intoxication. This occurs because citrate, which is used as an anticoagulant in stored blood, binds to calcium, leading to hypocalcemia.
Signs of citrate toxicity include:
- Muscle cramps and tetany (due to low calcium levels)
- Hypotension and arrhythmias
- Prolonged QT interval on ECG
To prevent citrate toxicity, calcium chloride is administered because it:
- Replenishes calcium levels that have been chelated by citrate.
- Prevents hypocalcemia-related complications, such as cardiac arrhythmias.
Thus, calcium chloride is the best choice to prevent citrate intoxication during massive transfusion.
Why Other Options Are Incorrect:
A. Sodium chloride – Incorrect because:
- Normal saline (0.9% NaCl) is used to maintain fluid balance, but it does not counteract citrate-induced hypocalcemia.
C. Lithium chloride – Incorrect because:
- Lithium has no role in preventing citrate intoxication and is mainly used in psychiatric conditions (e.g., bipolar disorder).
D. Potassium gluconate – Incorrect because:
- Potassium is important in electrolyte balance, but the main concern in citrate toxicity is calcium depletion, not potassium deficiency.
- Excess potassium could be dangerous, especially in patients receiving transfusions, as stored blood already has high potassium levels.
E. Magnesium sulfate – Incorrect because:
- Magnesium is important in neuromuscular function and cardiac stability, but it does not correct citrate-induced hypocalcemia.
Thus, calcium chloride is the most appropriate choice to prevent citrate intoxication during blood transfusion.
17 After spending her vacation at the seaside, 25-year-old woman developed complaints of weakness, myalgia, fever of 38.5°C, and pain and edema in her radiocarpal and elbow joints. Objectively, she has an erythematous rash on her cheeks and nasal dorsum and ulcers on her oral mucosa. Serological testing detects an increase in the titers of anti-nuclear antibodies and antibodies to native-DNA. What is the most likely diagnosis in this case?
A, Systemic scleroderma
B.Dermatomyositis
C. Rheumatoid arthritis
D. Systemic lupus erythematosus
E. Acute rheumatic fever
Correct Answer: D. Systemic lupus erythematosus (SLE)
Explanation:
This 25-year-old woman presents with symptoms suggestive of systemic lupus erythematosus (SLE), an autoimmune disorder that affects multiple organ systems. The key features in this case include:
- Fever, weakness, and myalgia → Common systemic symptoms of SLE.
- Polyarthritis (radiocarpal and elbow joint involvement) → SLE commonly causes a non-erosive, migratory arthritis.
- Erythematous rash on cheeks and nasal dorsum (“butterfly rash”) → A classic hallmark of SLE, which is photosensitive.
- Oral ulcers → Common in SLE.
- Positive serology: Anti-nuclear antibodies (ANA) and anti-double-stranded DNA (anti-dsDNA) → Highly specific for SLE.
SLE is often triggered or exacerbated by sun exposure, which aligns with this patient’s history of a vacation at the seaside.
Why Other Options Are Incorrect:
A. Systemic scleroderma – Incorrect because:
- Scleroderma is characterized by skin thickening, Raynaud’s phenomenon, and esophageal dysmotility, none of which are present here.
- It is associated with anti-centromere or anti-Scl-70 antibodies, rather than anti-dsDNA antibodies.
B. Dermatomyositis – Incorrect because:
- Dermatomyositis presents with proximal muscle weakness, heliotrope rash (periorbital discoloration), and Gottron’s papules (over joints).
- This patient has joint pain but no muscle weakness, and ANA and anti-dsDNA are more specific to SLE.
C. Rheumatoid arthritis (RA) – Incorrect because:
- RA causes symmetrical, erosive polyarthritis, primarily affecting the small joints of hands and feet.
- SLE arthritis is non-erosive, and RA does not typically present with a butterfly rash or oral ulcers.
- RA is associated with rheumatoid factor (RF) and anti-CCP antibodies, not anti-dsDNA.
E. Acute rheumatic fever (ARF) – Incorrect because:
- ARF typically follows streptococcal throat infection and presents with migratory arthritis, carditis, Sydenham’s chorea, and erythema marginatum.
- Anti-dsDNA and ANA are not markers of ARF.
Conclusion:
The presence of a butterfly rash, polyarthritis, oral ulcers, and positive ANA and anti-dsDNA antibodies makes systemic lupus erythematosus (SLE) the most likely diagnosis.
- An 89-year-old patient complains of dry hacking cough that occurs mainly in the morning. He has history of ischemic heart disease, stable angina pectoris, functional class II, and essential hypertension, stage 2, degree 2. The patient constantly takes lisinopril, bisoprolol, aspirin, and rosuvastatin. What drug has caused the cough in the patient?
A. Lisinopril
B. Acetylsalicylic acid
C.
D. Bisoprolol
E. Rosuvastatin
Correct Answer: A. Lisinopril
Explanation:
Lisinopril, an ACE inhibitor (Angiotensin-Converting Enzyme inhibitor), is well known for causing a dry hacking cough in some patients. This side effect is related to the accumulation of bradykinin, a peptide that can irritate the respiratory tract.
- The cough often occurs mainly in the morning, which is a common pattern.
- The mechanism involves bradykinin and substance P, which are normally degraded by ACE, but ACE inhibitors prevent this degradation, leading to increased levels of these irritants in the lungs.
- This side effect is dose-dependent and may persist as long as the patient is on the ACE inhibitor.
Why Other Options Are Incorrect:
B. Acetylsalicylic acid (Aspirin) – Incorrect because:
- Aspirin is an antiplatelet medication that is commonly used for cardiovascular protection.
- It does not cause a dry cough and would be unlikely to contribute to the symptoms described.
C. Bisoprolol – Incorrect because:
- Bisoprolol is a beta-blocker used for managing hypertension and angina.
- Beta-blockers typically do not cause a cough as a side effect.
D. Rosuvastatin – Incorrect because:
- Rosuvastatin is a statin, which is used to manage high cholesterol and prevent cardiovascular events.
- Statins do not cause a dry cough.
Thus, lisinopril is the most likely drug causing the dry hacking cough in this patient.
19. A 64-year-old patient with a tumor of the sigmoid colon and chronic thrombophlebitis of the deep veins of the right leg is scheduled for a surgery. What is the optimal medicine. for prevention of deep vein thrombosis in this patient?
A. Tranexamic acid
B. Acetylsalicylic acid
C. Clopidogrel
D. Regular heparin
E. Low-molecular-weight heparin
Correct Answer: E. Low-molecular-weight heparin
Explanation:
This patient is at high risk for deep vein thrombosis (DVT) due to the presence of a tumor of the sigmoid colon (which can increase the risk of clot formation) and chronic thrombophlebitis of the deep veins (which increases the risk of venous stasis and clot formation). For surgical patients with such risks, the optimal prevention of DVT involves the use of anticoagulants.
- Low-molecular-weight heparin (LMWH), such as enoxaparin, is the most appropriate choice in this scenario because it provides prophylactic anticoagulation, is easier to use than unfractionated heparin, and has a lower risk of bleeding.
- LMWH works by inactivating factor Xa and thrombin, preventing clot formation. It is commonly used in perioperative settings for patients at high risk of thromboembolism.
Why Other Options Are Incorrect:
A. Tranexamic acid – Incorrect because:
- Tranexamic acid is an antifibrinolytic agent that helps prevent the breakdown of clots.
- It is used to treat excessive bleeding or to prevent bleeding in certain procedures, but it does not prevent the formation of deep vein thrombosis.
B. Acetylsalicylic acid (Aspirin) – Incorrect because:
- Aspirin is an antiplatelet agent that inhibits platelet aggregation.
- While it can reduce the risk of arterial thrombosis, it is not effective for the prevention of venous thromboembolism (DVT and pulmonary embolism), which is more relevant in this patient with a tumor and thrombophlebitis.
C. Clopidogrel – Incorrect because:
- Clopidogrel is another antiplatelet agent, like aspirin, and works by inhibiting platelet aggregation.
- It is primarily used to prevent arterial thromboembolic events (such as in coronary artery disease or after stenting), but it does not provide adequate protection against venous thromboembolism like DVT.
D. Regular heparin – Incorrect because:
- Unfractionated heparin (regular heparin) is used in hospitalized patients, but it requires close monitoring of activated partial thromboplastin time (aPTT) and may be less convenient than LMWH.
- It is less commonly used for DVT prophylaxis in surgery compared to LMWH, especially for patients with a lower bleeding risk and those at high risk for thrombosis.
Thus, low-molecular-weight heparin (LMWH) is the most effective and convenient option for DVT prevention in this patient, particularly in the perioperative setting.
20. A 35-year-old patient developed general anxiety, brief agitation, dyspnea, and pain behind the sternum and in the lumbar region during the transfusion of 400 mL of packed erythrocytes. Objectively, the following is observed: cyanosis of the skin and mucosa, tachycardia 110/min, blood pressure 90/40 mm Hg. What is the most likely diagnosis in this case?
A. Massive transfusion syndrome
B. Pulmonary thromboembolism
C. Citrate intoxication
D. Anaphylactic shock
E. Hemotransfusion shock
Correct Answer: E. Hemotransfusion shock
Explanation:
The patient has developed symptoms shortly after the transfusion of 400 mL of packed erythrocytes, including anxiety, agitation, dyspnea, chest pain, cyanosis, tachycardia, and hypotension. These signs suggest an acute transfusion reaction leading to hemotransfusion shock, which can result from:
- Incompatibility of blood types (i.e., transfusion of the wrong blood group) leading to hemolysis of the transfused blood cells, causing the release of hemoglobin into the bloodstream.
- This can result in vasodilation, shock, cyanosis, and hypotension.
- Symptoms can also be exacerbated by the inflammatory response to the transfusion, leading to a systemic reaction.
Hemotransfusion shock is a rare but severe reaction to blood transfusions that involves vascular collapse and hypoxia. The timing of the reaction (during or shortly after transfusion) makes this diagnosis highly likely.
Why Other Options Are Incorrect:
A. Massive transfusion syndrome – Incorrect because:
- Massive transfusion syndrome refers to complications occurring from large volume transfusions, typically over a prolonged period (more than 10 units).
- This patient received only 400 mL of packed erythrocytes, which is relatively small in volume and not characteristic of massive transfusion syndrome.
B. Pulmonary thromboembolism – Incorrect because:
- Pulmonary thromboembolism (PE) can cause dyspnea, cyanosis, tachycardia, and hypotension, but it is less likely to be associated with the timing of the transfusion.
- PE usually arises from a deep vein thrombosis (DVT) or fat embolism and would not be caused directly by transfusion.
C. Citrate intoxication – Incorrect because:
- Citrate intoxication occurs when citrate, used as an anticoagulant in stored blood, binds to calcium and causes hypocalcemia.
- The typical symptoms of citrate toxicity include muscle cramps, tetany, and arrhythmias, not the acute cardiovascular collapse observed in this patient.
- The timing and clinical features of this case suggest a more acute, transfusion-related reaction, rather than citrate intoxication.
D. Anaphylactic shock – Incorrect because:
- Anaphylactic shock is an allergic reaction that can cause hypotension, dyspnea, and cyanosis, but typically involves a history of allergies and a rapid onset of symptoms immediately after exposure to an allergen.
- Anaphylaxis in transfusion reactions is more common in patients with a known history of allergies to transfused proteins or blood products, but it would be less likely in this context without prior allergy history.
Thus, hemotransfusion shock is the most likely diagnosis based on the patient’s acute symptoms following transfusion, including dyspnea, cyanosis, tachycardia, and hypotension.
21. Several chemical substances enter the human body from atmospheric air. What is the type of joint action, where the overall effect on the human body is more than the sum of the individual effects of each separate substance included in the combination?
A. Combined action
B. Isolated action
C. Complex action
D. Potentiation
E. Antagonism
Correct Answer: D. Potentiation
Explanation:
Potentiation refers to a type of joint action where the combined effect of two or more substances is greater than the sum of their individual effects. In other words, when two or more substances are present together, they enhance each other’s effects, leading to a stronger overall response than would be expected if the substances acted separately.
- Example: One substance might increase the absorption or effectiveness of another, causing the combined effect to be more potent than the individual effects.
Why Other Options Are Incorrect:
A. Combined action – Incorrect because:
- Combined action refers to the additive effects of different substances when used together. The effects of the substances in combined action are simply the sum of their individual effects, not greater.
- This is different from potentiation, where the total effect is greater than the sum of individual effects.
B. Isolated action – Incorrect because:
- Isolated action refers to the individual effects of a substance when it acts alone.
- This does not describe the enhanced effect seen when two substances are used together.
C. Complex action – Incorrect because:
- Complex action typically refers to a combination of different mechanisms or effects in the body that are more intricate, but it does not specifically imply an enhanced or synergistic effect, as potentiation does.
E. Antagonism – Incorrect because:
- Antagonism refers to the situation where one substance reduces or opposes the effect of another.
- This is the opposite of potentiation, where substances work together to enhance each other’s effects.
Thus, potentiation best describes the scenario where the combined effect is greater than the sum of the individual effects.
22. An 8-year-old boy has a circular spot under 1.0 cm in diameter on his scalp. Objectively, the skin in the area of the spot is pink and covered in small flour-like scales, the hair there is broken at the height of 4-5 mm from the skin surface. The patient feels no subjective sensati- ons there. His sister has a similar spot. What is the most likely diagnosis in this case?
A. Lupus erythematosus
B. Microsporia
C. Psoriasis
D. Alopecia areata
E. Scleroderma
Correct Answer: B. Microsporia
Explanation:
The patient’s presentation is highly suggestive of microsporia, a fungal infection of the scalp caused by dermatophytes (primarily Microsporum species). Key features supporting this diagnosis include:
- Circular spot with a diameter of 1.0 cm on the scalp, which is a common presentation for a dermatophyte infection.
- Pink skin with small flour-like scales, which are characteristic of ringworm infections of the scalp (tinea capitis).
- Broken hair at a height of 4-5 mm from the skin surface, which is typical of fungal infections that weaken the hair shaft.
- Asymptomatic, with no pain or itching, which is often seen in microsporia.
- The fact that the sister has a similar spot suggests a family history of fungal infection, which is common in dermatophyte infections like microsporia.
Why Other Options Are Incorrect:
A. Lupus erythematosus – Incorrect because:
- Lupus erythematosus usually presents with a discoid rash (a red, scaly, circular patch), but this typically involves more inflammation, and would not be associated with broken hair.
- The absence of subjective symptoms and the scaly appearance makes lupus less likely.
C. Psoriasis – Incorrect because:
- Psoriasis is typically characterized by well-demarcated, silvery, scaly plaques, often on the scalp or elbows.
- It would not cause broken hair at the follicle level in the manner described in this case.
D. Alopecia areata – Incorrect because:
- Alopecia areata is characterized by well-defined, round patches of hair loss with no scaling.
- In this case, the scaling and broken hair are more indicative of a fungal infection rather than an autoimmune hair loss condition like alopecia areata.
E. Scleroderma – Incorrect because:
- Scleroderma involves tightening of the skin due to excessive collagen deposition, which is not present here.
- The scaly patches and broken hair are not typical features of scleroderma.
Thus, the most likely diagnosis for this 8-year-old boy with a circular scaly patch on the scalp and broken hair is microsporia, a fungal infection.
23 A trial was conducted to study the effectiveness of arterial hypertension treatment using a new drug, compared with traditional therapy. The were 3,000 participants in the trial, di- vided into two identical groups, taking into account their age, sex, stage of the disease, etc. One group was receiving treatment with the new drug, while the other was receiving traditional therapy and was the control group. The trial participants did not know which therapy they were receiving. What type of trial was conducted in this case?
A. Blind randomized controlled trial
B. Case-control study
C. Prospective cohort study
D. Cross-sectional study
E. Double-blind randomized controlled trial
Correct Answer: E. Double-blind randomized controlled trial
Explanation:
In this scenario, the trial is designed as a randomized controlled trial (RCT) with a double-blind approach. The key features that support this conclusion are:
- Randomized: Participants were divided into two identical groups using a random method, ensuring that there is no bias in how they were assigned to treatment (new drug vs. traditional therapy).
- Controlled: One group is receiving traditional therapy (control group), and the other group is receiving the new drug.
- Double-blind: The participants did not know which therapy they were receiving, which suggests that both the participants and the researchers are unaware of the group assignments. This reduces the risk of bias in reporting results and evaluating outcomes.
A double-blind method enhances the validity and reliability of the results by preventing expectations or biases from influencing the study outcomes.
Why Other Options Are Incorrect:
A. Blind randomized controlled trial – Incorrect because:
- While this describes a randomized controlled trial, it does not specify that both participants and researchers are unaware of the group assignments. A “blind” trial typically refers to only one group being unaware (either the participants or the researchers), whereas double-blind means both groups are unaware.
B. Case-control study – Incorrect because:
- A case-control study is observational, where participants are selected based on the presence (cases) or absence (controls) of a particular outcome (e.g., disease). This is different from a randomized controlled trial, which is an interventional study.
C. Prospective cohort study – Incorrect because:
- A prospective cohort study involves following two or more groups (cohorts) over time based on their exposure to a certain factor or treatment. Participants are not randomly assigned to treatment, and the study is typically observational, not interventional.
D. Cross-sectional study – Incorrect because:
- A cross-sectional study assesses a population at one point in time to determine the prevalence of a condition or characteristic. It does not involve random assignment or treatments over time.
Thus, the double-blind randomized controlled trial is the correct description of this study, as it involves random assignment, control groups, and blinding of both the participants and researchers.
24. A 2-year-old child has abdominal distension and constipations. According to the patient’s medical history, these symptoms first appeared at the age of nine months. Irrigography detects a narrowed arca in the distal segments of the large intestine with a funnel- like transition into a suprastenotic expansion. What is the most likely diagnosis in this case?
A. Hirschsprung’s disease
B. Intestinal duplication
C. Crohn’s disease
D. Dolichosigma
E. Meckel’s diverticulum
Correct Answer: A. Hirschsprung’s disease
Explanation:
The patient presents with abdominal distension, constipation, and narrowed areas in the distal large intestine as seen on irrigography, which suggests a structural abnormality in the bowel. The key features in the case that support Hirschsprung’s disease are:
- The onset of symptoms at 9 months of age, which is typical for Hirschsprung’s disease, as it often presents in infants or young children with chronic constipation and abdominal distension.
- The narrowing of the distal segments of the large intestine and the funnel-like transition into a suprastenotic expansion seen on irrigography is a characteristic finding. In Hirschsprung’s disease, there is an absence of ganglion cells in the distal colon, leading to functional obstruction and dilated bowel segments proximal to the affected area.
Why Other Options Are Incorrect:
B. Intestinal duplication – Incorrect because:
- Intestinal duplications are congenital malformations that can cause symptoms like abdominal pain and obstruction, but they typically present differently, often as a sac-like structure on imaging, not with the characteristic narrowing and funneling seen in this case.
- These duplications usually occur in the small intestine rather than the distal colon.
C. Crohn’s disease – Incorrect because:
- Crohn’s disease is an inflammatory bowel disease that can cause symptoms such as abdominal pain, diarrhea, and weight loss. While it can cause narrowing or strictures, it typically affects the small intestine or ileocolonic junction and is associated with inflammation, not a structural narrowing due to absent ganglion cells.
- The onset at 9 months is not typical for Crohn’s disease, which usually presents later in childhood or adulthood.
D. Dolichosigma – Incorrect because:
- Dolichosigma refers to an abnormally elongated sigmoid colon that can cause constipation and abdominal distension. While it can cause symptoms similar to those seen in Hirschsprung’s disease, irrigography would typically show a longer, more tortuous sigmoid colon, rather than the characteristic funnel-shaped narrowing and dilated proximal colon seen in Hirschsprung’s disease.
E. Meckel’s diverticulum – Incorrect because:
- Meckel’s diverticulum is a congenital abnormality in the small intestine, typically presenting with pain, bleeding, or obstruction, but it does not cause the narrowing and funnel-like transitions seen in this case.
- It is also less likely to present with the chronic constipation and abdominal distension seen in this child.
Thus, Hirschsprung’s disease is the most likely diagnosis, given the age of onset, chronic constipation, abdominal distension, and the characteristic imaging findings of narrowing and dilation in the distal colon.
25. A 47-year-old patient, a designer, developed complaints of palpitations, excessive sweating, hand tremor, and pain in the eyes. According to the patient’s medical history, she has been taking amiodarone, warfarin, lisinopril, torasemide, and trimetazidine for 7 months for treatment of her persistent atrial fibrillation. What drug has most likely caused the deterioration of the patient’s condition?
A. Lisinopril
B. Warfarin
C. Trímetazidine
D. Torasemide
E. Amiodarone
Correct Answer: E. Amiodarone
Explanation:
The patient’s symptoms of palpitations, excessive sweating, hand tremor, and pain in the eyes suggest a hyperthyroid or thyroid-related issue, particularly since amiodarone is known to affect thyroid function.
- Amiodarone is an antiarrhythmic drug that is well known to cause thyroid dysfunction, including both hypothyroidism and hyperthyroidism. One of the side effects of amiodarone is amiodarone-induced thyrotoxicosis (AIT), a condition that can present with symptoms like palpitations, sweating, tremor, and eye discomfort. This is most often due to the iodine content of amiodarone, which affects thyroid function.
- In this case, the combination of these symptoms in the context of taking amiodarone for a prolonged period (7 months) makes it the most likely cause of the patient’s symptoms.
Why Other Options Are Incorrect:
A. Lisinopril – Incorrect because:
- Lisinopril is an ACE inhibitor commonly used for treating hypertension and heart failure. While it can cause side effects such as hyperkalemia or renal dysfunction, it is not typically associated with symptoms of hyperthyroidism like tremors, sweating, or eye pain.
B. Warfarin – Incorrect because:
- Warfarin is an anticoagulant used to prevent blood clots. While it requires monitoring due to its risk of bleeding and interactions with other drugs, it does not cause symptoms like palpitations, tremors, or eye pain. It is unlikely to be the cause of this patient’s symptoms.
C. Trimetazidine – Incorrect because:
- Trimetazidine is a metabolic agent used for managing angina. It does not typically cause hyperthyroid symptoms. While trimetazidine can cause side effects like dizziness or gastrointestinal issues, it is not known to cause palpitations, tremor, or eye pain associated with thyroid dysfunction.
D. Torasemide – Incorrect because:
- Torasemide is a loop diuretic used to treat fluid retention in conditions like heart failure. Its main side effects include electrolyte disturbances (e.g., hypokalemia), but it is not typically associated with hyperthyroid symptoms like tremor, sweating, and eye pain.
Thus, the most likely drug causing the deterioration of the patient’s condition is amiodarone, due to its well-established link with thyroid dysfunction, specifically amiodarone-induced thyrotoxicosis.
26. A 22-year-old patient lies in bed with his head raised high and feels no discomfort in this position. He enters the conversation reluctantly, responds to whispered speech, gives one-word answers. His face is indifferent and hypomimic, the forehead is wrinkled, the lips are are puckered. The patient moves very little and often freezes for a long time in an uncomfortable position. This condition has developed gradually over the course of a week with no apparent cause. What condition is observed in the patient?
A. Exogenous stupor
B. Catatonic substupor
C. Apathetic substuporpor
D. Depressive substupor
E. Psychogenic stupor
Correct Answer: B. Catatonic substupor
Explanation:
The patient’s presentation, with reluctance to engage in conversation, one-word answers, a hypomimic face, pursed lips, and freezing in uncomfortable positions, is indicative of a catatonic state, specifically catatonic substupor. Here’s why:
- Catatonia is a motor and behavioral syndrome that involves marked changes in movement, behavior, and speech. It can present with symptoms like immobility, mutism, and posturing (e.g., freezing in a position for extended periods).
- Catatonic substupor refers to a milder form of catatonia, where the patient is still conscious but exhibits signs of decreased motor activity, apathy, and minimal verbal response (e.g., one-word answers).
- The gradual onset over a week with no apparent cause suggests a psychiatric disorder, and the absence of discomfort when the patient is positioned with his head raised suggests that there are no significant organic causes (e.g., neurological issues).
Why Other Options Are Incorrect:
A. Exogenous stupor – Incorrect because:
- Exogenous stupor refers to a stuporous state caused by external factors, such as substance intoxication or toxins, which the patient does not demonstrate in this case. The gradual onset without a clear external cause makes exogenous stupor less likely.
C. Apathetic substupor – Incorrect because:
- Apathetic substupor involves lack of motivation or emotional expression but does not typically include catatonic features such as motor immobility, posturing, or freezing. The patient exhibits more motoric immobility than what would be seen in an apathetic state.
D. Depressive substupor – Incorrect because:
- While depression can involve hypomimia and lack of engagement, depressive substupor typically presents with profound sadness and lack of energy, but it does not usually involve the specific motor behaviors (like freezing in uncomfortable positions) seen in catatonia.
E. Psychogenic stupor – Incorrect because:
- Psychogenic stupor refers to a complete lack of responsiveness or immobility due to psychological causes (e.g., trauma or stress), but this patient is still responding, albeit minimally, which points more to catatonic substupor rather than psychogenic stupor.
Thus, the most appropriate diagnosis for this patient’s gradual onset of motor immobility, hypomimia, and minimal verbal response is catatonic substupor.
27. A 65-year-old patient complains of frequent painful urination, moderate weight loss observed over the last 3-4 months, and massi- ve macrohematuria with excretion of shapeless blood clots. According to the patient’s medi- cal history, macrohematuria first appeared 3 months ago with no apparent cause and was not accompanied by pain and dysuria back then, a few days later the bleeding stopped on its own. What is the most likely diagnosis in this case?
A. Chronic cystitis
B. Bladder tumor
C. Acute cystitis
D. Urolithiasis
E. Kidney tumor
Correct Answer: B. Bladder tumor
Explanation:
The patient’s symptoms, including painless macrohematuria (gross hematuria) that initially resolved on its own, followed by progression to dysuria, weight loss, and recurrent bleeding, are highly suggestive of bladder cancer.
- Bladder tumors, particularly urothelial carcinoma, are the most common cause of painless hematuria in older adults.
- The initial episode of hematuria without pain, which later developed into painful urination and weight loss, suggests tumor progression.
- Shapeless blood clots indicate a bleeding source from the bladder rather than the kidneys, where clots tend to be more uniform and cylindrical.
Risk factors for bladder cancer include smoking, exposure to industrial chemicals (aniline dyes, aromatic amines), chronic bladder irritation, and age >60 years.
Why Other Options Are Incorrect:
A. Chronic cystitis – Incorrect because:
- Chronic cystitis typically presents with urinary frequency, dysuria, and suprapubic discomfort, but it rarely causes massive macrohematuria with blood clots.
- Weight loss is not characteristic of cystitis.
C. Acute cystitis – Incorrect because:
- Acute cystitis (bladder infection) presents with dysuria, frequency, urgency, and suprapubic pain, but gross hematuria with blood clots is uncommon.
- Additionally, hematuria in acute cystitis is usually microscopic, not massive.
D. Urolithiasis – Incorrect because:
- Bladder stones can cause hematuria and dysuria, but they usually cause severe colicky pain (renal colic), which was not present in this patient.
- Blood clots in urolithiasis are often smaller and do not typically cause prolonged hematuria.
E. Kidney tumor – Incorrect because:
- Kidney tumors (e.g., renal cell carcinoma) can cause hematuria, but the blood clots tend to be elongated and worm-like (molded by the ureters), unlike the shapeless clots seen in this case.
- Additionally, kidney tumors are more commonly associated with flank pain and a palpable mass, which were not mentioned in this patient.
Conclusion:
The combination of painless macrohematuria (initially), later development of painful urination, weight loss, and blood clots is highly suggestive of a bladder tumor (most likely urothelial carcinoma).
28. A 58-year-old patient complains of dizziness, diplopia, and disturbed gait. The patient has history of three similar episodes. Objectively, the following is observed: horizontal nystagmus, diplopia, ataxia on the right in the Romberg’s position. The patient performs coordination tests unsteadily on the right. Three hours later, the neurological symptoms regressed. What is the most likely diagnosis in this case?
A. Transient ischemic attack
B. Ischemic stroke
C. Subarachnoid hemorrhage
D. Multiple sclerosis
E. Brain tumor
Correct Answer: A. Transient ischemic attack (TIA)
Explanation:
The patient presents with transient neurological symptoms including dizziness, diplopia, nystagmus, ataxia, and gait disturbances, which resolved within three hours. These features are highly suggestive of a transient ischemic attack (TIA), particularly one affecting the vertebrobasilar circulation.
- TIA is a temporary, reversible neurological event caused by transient ischemia (lack of blood flow) to a specific brain region, without permanent infarction.
- The presence of recurrent similar episodes further supports TIA, as multiple transient attacks are common before an eventual stroke.
- Symptoms localized to the posterior circulation (brainstem, cerebellum) suggest vertebrobasilar insufficiency, which can cause nystagmus, ataxia, and diplopia.
Key diagnostic clues:
✔ Sudden onset of focal neurological deficits
✔ Complete resolution within a few hours (usually <24 hours)
✔ Multiple past episodes suggest ongoing vascular insufficiency
Why Other Options Are Incorrect:
B. Ischemic stroke – Incorrect because:
- Stroke symptoms persist longer than 24 hours and usually do not fully resolve within a few hours.
- TIAs are warning signs for future strokes, but this case does not meet the criteria for a full stroke due to the rapid resolution of symptoms.
C. Subarachnoid hemorrhage – Incorrect because:
- SAH typically presents with a sudden, severe “thunderclap” headache, often described as “the worst headache of my life,” which is absent here.
- SAH symptoms do not resolve quickly; instead, they worsen over time.
D. Multiple sclerosis (MS) – Incorrect because:
- MS typically causes chronic, progressive, or relapsing-remitting neurological symptoms that persist for days to weeks, not hours.
- MS lesions affect different parts of the CNS over time, leading to a varied distribution of symptoms rather than the transient, localized symptoms seen in TIA.
E. Brain tumor – Incorrect because:
- Brain tumors cause progressive symptoms that worsen over weeks to months, rather than sudden-onset, transient episodes.
- Symptoms from a tumor would not fully resolve within a few hours.
Conclusion:
The sudden onset and complete resolution of neurological deficits within a few hours, combined with a history of similar episodes, strongly suggest a transient ischemic attack (TIA), likely due to vertebrobasilar insufficiency. This condition is a warning sign for an impending stroke and requires urgent evaluation and management to prevent a full ischemic stroke.
29. A 20-year-old patient took 50 sleeping pi- is, attempting to commit suicide. Objectively, the patient’s condition is severe, the pupiIs are dilated and unresponsive to light. The patient’s breathing is hoarse, noisy at a di- stance, respiratory rate 48/min, auxiliary muscles are involved in the act of breathing. Marked cyanosis is observed. Auscultation detects numerous wet crackles in the lungs, Pulse 126/min, shallow, blood pressure -85/45 mm Hg. What aid must be provided to the patient first?
A. Gastric lavage
B. Hyperbaric oxygenation
C. Forced diuresis
D. Tracheal intubation and artificial lung ventilation
E. Toilet of the tracheobronchial tree
Correct Answer: D. Tracheal intubation and artificial lung ventilation
Explanation:
The patient presents with severe respiratory distress, unresponsiveness, cyanosis, and aspiration signs, indicating respiratory failure secondary to drug overdose. The first priority in managing this patient is to secure the airway through tracheal intubation and mechanical ventilation to prevent respiratory arrest and hypoxic brain injury.
- Key signs of impending respiratory failure in this case:
- Severe respiratory distress (RR = 48/min, use of accessory muscles, noisy breathing)
- Cyanosis (suggesting hypoxia and inadequate oxygenation)
- Unresponsive, dilated pupils (suggesting central nervous system depression due to overdose)
- Pulmonary congestion with crackles (suggesting aspiration or non-cardiogenic pulmonary edema)
- Hypotension (BP = 85/45 mm Hg) and tachycardia (126/min), indicating shock
🚨 Immediate intervention with airway management (intubation and ventilation) is necessary to prevent fatal respiratory failure.
Why Other Options Are Incorrect:
A. Gastric lavage – Incorrect because:
- While gastric lavage is useful for drug overdose, it is not the first priority in a patient with severe respiratory distress and altered mental status.
- Aspiration risk is high in an unresponsive patient without a secured airway.
- Lavage is most effective within 1 hour of ingestion; after this, drug absorption is usually complete.
B. Hyperbaric oxygenation – Incorrect because:
- Hyperbaric oxygen therapy is primarily used for carbon monoxide poisoning or severe hypoxia but is not a first-line treatment for drug overdose-induced respiratory failure.
C. Forced diuresis – Incorrect because:
- Forced diuresis is not a priority in this emergency. It may be considered after stabilization in cases of toxic ingestion of renally excreted drugs, but the primary concern here is airway protection and oxygenation.
E. Toilet of the tracheobronchial tree – Incorrect because:
- While clearing secretions may be necessary, it does not address the primary issue of inadequate ventilation.
- This should be performed after airway protection (intubation), not as the first step.
Conclusion:
🚑 Tracheal intubation and mechanical ventilation are the most urgent life-saving interventions in this patient with severe respiratory depression, cyanosis, and aspiration risk due to drug overdose. Other treatments, such as gastric lavage or toxin elimination, can be considered only after airway protection is ensured.
30. A 36-year-old patient developed an attack of acute arthritis in the first metatarsophalangeal joint of the right foot after drinking alcohol and eating fatty food the night before. Objectively, the first toe of the right foot is edematous, the skin there is cyanotic and hot to the touch. Complete blood count: leukocytes 12 109/L, left shift, ESR 42 mm/hour. What drug should be prescribed first in this case?
A. Prednisolone
B. Paracetamol
C. Ibuprofen
D. Allopurinol
E Colchicine
Correct Answer: E. Colchicine
Explanation:
The patient presents with acute monoarthritis of the first metatarsophalangeal (MTP) joint, which is highly suggestive of acute gouty arthritis, especially given the history of alcohol and fatty food consumption, both of which are known gout triggers.
🔹 Key features supporting acute gout:
- Sudden onset of severe joint pain
- Involvement of the first MTP joint (“podagra”)
- Red, swollen, hot, and tender joint
- Elevated WBC count (12 × 10⁹/L) and ESR (42 mm/hr), indicating inflammation
📌 Colchicine is the first-line treatment for acute gout attacks because it effectively reduces inflammation by inhibiting neutrophil migration and cytokine release within hours. It is most effective when given within the first 24 hours of an attack.
💊 Colchicine dosing for acute gout:
- 1.2 mg initially, followed by 0.6 mg in 1 hour
- Then 0.6 mg once or twice daily for a few days
Why Other Options Are Incorrect:
A. Prednisolone – Incorrect because:
- Corticosteroids (e.g., prednisolone 30-40 mg daily) are an option for severe cases or when NSAIDs and colchicine are contraindicated (e.g., renal failure, GI ulcers).
- However, colchicine is preferred as first-line therapy unless contraindicated.
B. Paracetamol – Incorrect because:
- Paracetamol (acetaminophen) has no anti-inflammatory properties and is ineffective for treating acute gout attacks.
- NSAIDs or colchicine are preferred.
C. Ibuprofen – Incorrect because:
- NSAIDs (e.g., ibuprofen, naproxen, indomethacin) are also first-line choices, but colchicine is preferred if given early.
- NSAIDs may be a good alternative, especially if colchicine is not well tolerated.
D. Allopurinol – Incorrect because:
- Allopurinol is used for chronic urate-lowering therapy but should NOT be started during an acute gout attack.
- Initiating allopurinol during an acute attack can worsen symptoms by mobilizing urate crystals.
- It should be introduced only after the attack has fully resolved (2-4 weeks later).
Conclusion:
Colchicine is the first-line treatment for acute gouty arthritis due to its rapid anti-inflammatory effects, particularly when given within the first 24 hours. NSAIDs (e.g., ibuprofen) or corticosteroids (e.g., prednisolone) may be alternatives, but allopurinol should be avoided during an acute attack as it can worsen symptoms.
31. A 21-year-old patient complains of an itchy rash that appeared in the morning. The rash consisted of small blisters that disappeared after a few hours, leaving spots. According to the patient’s history, the day before she was staying for a long time outdoors, among flowering plants. Objectively, her general condition is satisfactory, erythematous spots with infiltration can be visualized in the area of her forehead, eyes, upper and lower limbs. What is the most likely diagnosis in this case?
A. Urticaria
B. Toxicoderma
C. Toxic allergic bullous epidermal necrolysis
D. Erythema multiforme exudativum
E. Erythroderma
Correct Answer: A. Urticaria
Explanation:
The patient’s symptoms are highly suggestive of urticaria, which is a common allergic reaction characterized by transient, itchy wheals (hives) that resolve within hours without scarring.
🔹 Key features supporting urticaria:
- Acute onset of an itchy rash
- Blisters that disappeared within hours, leaving only spots
- History of outdoor exposure to flowering plants, suggesting an allergic trigger (e.g., pollen, insect bites, or contact allergens)
- No systemic involvement or severe symptoms
📌 Urticaria occurs due to histamine release from mast cells, leading to pruritic wheals and erythema. It can be triggered by allergens, infections, stress, or even physical stimuli (e.g., cold, heat, pressure).
💊 Treatment includes:
- Oral antihistamines (e.g., cetirizine, loratadine, fexofenadine)
- Avoiding known allergens
- Corticosteroids (only if severe or persistent symptoms occur)
Why Other Options Are Incorrect:
B. Toxicoderma – Incorrect because:
- Toxicoderma (drug eruption) is usually caused by medications and presents as a generalized rash, often with systemic symptoms.
- The patient has no history of drug intake or systemic signs of drug reaction.
C. Toxic allergic bullous epidermal necrolysis (TEN) – Incorrect because:
- TEN is a life-threatening condition that presents with extensive skin necrosis, bullae, and systemic involvement (e.g., fever, malaise).
- It usually involves mucosal surfaces, which is not present in this patient.
D. Erythema multiforme exudativum – Incorrect because:
- Erythema multiforme (EM) presents with target lesions (central dusky area, surrounding red ring), typically triggered by viral infections (e.g., HSV) or medications.
- The rash lasts for several days to weeks, not just a few hours like in urticaria.
E. Erythroderma – Incorrect because:
- Erythroderma is a severe, widespread skin condition involving generalized redness, scaling, and systemic symptoms.
- The patient’s rash is localized and transient, which does not match erythroderma.
Conclusion:
The transient nature of the rash, its itchiness, and the history of allergen exposure strongly indicate acute urticaria. Treatment involves antihistamines and allergen avoidance, with corticosteroids only if symptoms are severe.
32. A 75-year-old woman complains of a tumor in her right breast. In her left mammary gland, palpation detects no pathology. In her right mammary gland, a lumpy, dense, painless, non-mobile formation was detected. The skin above the formation resembles a “lemon rind”. The nipple is retracted. Axillary, subclavian, and supraclavicular lymph nodes are not palpable. What pathological condition has most likely developed in the right mammary gland of the patient?
A. Diffuse cystic mastopathy
B. Malignant neoplasm
C. Solitary cyst
D. Tuberculosis
E. Fibroadenoma
Correct Answer: B. Malignant neoplasm
Explanation:
The clinical features described strongly suggest breast cancer, a malignant neoplasm of the breast.
🔹 Key findings supporting malignancy:
- Hard, non-mobile, and painless breast mass → Suggestive of an invasive tumor
- Skin changes (“lemon rind” appearance) → Indicates lymphatic involvement (peau d’orange)
- Nipple retraction → Common in invasive ductal carcinoma due to fibrosis pulling the nipple inward
- No palpable axillary, subclavian, or supraclavicular lymph nodes → Early-stage disease, but malignancy cannot be ruled out based on this
📌 Breast cancer is most common in older women (>50 years), making this 75-year-old woman highly susceptible.
🔬 Diagnosis Confirmation:
- Mammography & Ultrasound: To characterize the mass
- Core needle biopsy: To confirm malignancy and determine histological type
- MRI & PET scan: For staging in suspected advanced cases
💊 Treatment:
- Surgical excision (lumpectomy or mastectomy)
- Adjuvant therapy (chemotherapy, radiotherapy, hormone therapy based on receptor status)
Why Other Options Are Incorrect:
A. Diffuse cystic mastopathy (Fibrocystic Breast Disease) – Incorrect because:
- Typically presents as bilateral, tender, mobile nodular masses that fluctuate with the menstrual cycle.
- No peau d’orange or nipple retraction is seen.
C. Solitary cyst – Incorrect because:
- Breast cysts are fluid-filled, mobile, and often tender.
- They do not cause nipple retraction or peau d’orange.
D. Tuberculosis – Incorrect because:
- Rare in the breast but presents as chronic abscesses or sinus formation.
- Accompanied by systemic symptoms (fever, weight loss, night sweats), which are absent here.
E. Fibroadenoma – Incorrect because:
- Benign, well-circumscribed, mobile, painless mass that does not cause skin or nipple changes.
- Common in young women (<35 years), unlike this elderly patient.
Conclusion:
The non-mobile, hard mass, skin retraction (peau d’orange), and nipple inversion strongly indicate breast cancer. Immediate biopsy and imaging are necessary for confirmation.
33. A 64-year-old man complains of the absence of urination during the last 12 hours. Objectively, a painful mass is palpable above the pubis, the upper edge of the mass reaches the navel. Rectal examination detects an enlarged prostate. The prostate is painless, lumpy and has a stone-like density. Blood levels of prostate-specific antigen are 24.2 ng/mL. What is the cause of the acute urinary retention in this case?
A. Benign prostatic hyperplasia
B. Epididymo-orchitis
C. Prostate cancer
D. Chronic paraproctitis
E. Acute prostatitis
Correct Answer: C. Prostate cancer
Explanation:
The patient’s acute urinary retention (AUR) is caused by prostate cancer, as suggested by:
🔹 Key findings supporting prostate cancer:
- Enlarged, hard, lumpy, stone-like prostate → Suggestive of malignancy
- Markedly elevated prostate-specific antigen (PSA) (24.2 ng/mL) → Strongly indicates prostate cancer (>10 ng/mL is highly suspicious)
- Acute urinary retention with a large, painful suprapubic mass → Likely distended bladder due to obstruction
- Older age (64 years) → Prostate cancer is more common in older men
📌 Prostate cancer causes urinary retention by compressing the urethra, blocking urine flow.
🔬 Diagnosis Confirmation:
- Digital rectal exam (DRE): Hard, irregular prostate
- PSA levels: >4 ng/mL is suspicious; >10 ng/mL strongly suggests malignancy
- Transrectal ultrasound (TRUS) & MRI: To evaluate tumor extent
- Prostate biopsy: Confirms diagnosis
💊 Treatment:
- Localized cancer: Radical prostatectomy, radiation therapy
- Advanced cancer: Androgen deprivation therapy (ADT), chemotherapy
Why Other Options Are Incorrect:
A. Benign prostatic hyperplasia (BPH) – Incorrect because:
- BPH causes a smooth, symmetrically enlarged prostate, not a hard, lumpy, stone-like gland.
- BPH can cause urinary retention, but PSA levels are usually mildly elevated (<10 ng/mL), not as high as 24.2 ng/mL.
B. Epididymo-orchitis – Incorrect because:
- Presents with scrotal pain, swelling, and fever, not prostate enlargement or urinary retention.
D. Chronic paraproctitis – Incorrect because:
- A perianal infection with abscess formation, causing perianal pain and pus discharge, not urinary retention.
E. Acute prostatitis – Incorrect because:
- Presents with fever, chills, painful urination, perineal pain, and a tender, boggy prostate.
- PSA can be elevated, but not typically to 24.2 ng/mL.
Conclusion:
The hard, lumpy prostate, markedly elevated PSA, and urinary retention strongly suggest prostate cancer, which requires urgent evaluation and management.
- A 6-year-old child became acutely ill and developed fever, headache, and pain during swallowing. Three hours later, a bright-red fi- ne punctate rash appeared on the hyperemic skin. The rash is more numerous on the lateral surfaces of the body and in the natural folds. The oropharyngeal mucosa has a clearly demarcated hyperemia. There is a purulent plaque on the tonsils. What disease has developed in the child?
A. Measles
B. Diphtheria
C. Infectious mononucleosis
D. Rubella
E Scarlet fever
Correct Answer: E. Scarlet fever
Explanation:
Scarlet fever is a bacterial infection caused by group A Streptococcus (GAS) and is characterized by:
🔹 Key findings in this child:
- Fever, headache, sore throat → Classic early symptoms
- Bright red, fine punctate rash (“sandpaper-like”)
- Rash predominance in skin folds (Pastia’s lines)
- “Strawberry tongue” and oropharyngeal hyperemia
- Purulent plaque on tonsils → Suggestive of streptococcal pharyngitis
- Rapid progression (rash appearing within hours)
🔬 Diagnosis Confirmation:
- Throat culture for Streptococcus pyogenes
- Rapid antigen detection test (RADT) for GAS
- ASO (antistreptolysin O) titer (helpful for past infections)
💊 Treatment:
- Penicillin or amoxicillin (first-line treatment)
- Macrolides (e.g., azithromycin) for penicillin-allergic patients
Why Other Options Are Incorrect:
A. Measles – Incorrect because:
- Measles rash is maculopapular, begins on the face, and spreads downward.
- Associated with Koplik spots (white spots on the buccal mucosa).
- Rash appears after 3–4 days of high fever, not within 3 hours like in this case.
B. Diphtheria – Incorrect because:
- Presents with thick grayish-white pseudomembranes in the throat, not purulent plaques.
- Causes “bull neck” appearance due to cervical lymphadenopathy.
- Rash is not characteristic.
C. Infectious mononucleosis – Incorrect because:
- Caused by Epstein-Barr virus (EBV), presents with prolonged fever, lymphadenopathy, and splenomegaly.
- Tonsillar enlargement with white-gray exudate, but no fine rash.
D. Rubella – Incorrect because:
- Rash is maculopapular, starts on the face, and spreads quickly downward.
- No purulent tonsillar plaques or severe pharyngitis.
- Mild illness with low-grade fever, unlike the high fever in this child.
Conclusion:
This child’s acute onset of fever, sore throat, fine punctate rash (especially in skin folds), and tonsillar exudate strongly suggest scarlet fever, which requires antibiotic treatment to prevent rheumatic fever and other complications.
- During examination, the patient presents with a lacerated wound in the area of the right lower leg. Dark blood constantly flows from the wound. What emergency aid should be provided in this case?
A. Immobilize the limb
B. Apply cold to the wound
C Apply a pressure bandage
D. Apply a tourniquet below the wound
E. Apply a tourniquet above the wound
Correct Answer: C. Apply a pressure bandage
Explanation:
The continuous dark blood flow from the wound suggests venous bleeding, as venous blood is typically dark red and flows steadily (unlike arterial bleeding, which is bright red and spurts with each heartbeat).
The most appropriate first aid for venous bleeding is to apply a pressure bandage, which helps compress the injured vessels, slow blood flow, and promote clot formation. A pressure bandage is effective in controlling venous hemorrhage without cutting off circulation, which can happen with a tourniquet.
Why Other Options Are Incorrect:
A. Immobilize the limb:
Immobilization helps prevent further injury but does not stop active bleeding.
It is useful for fractures but is not the first priority in active bleeding.
B. Apply cold to the wound:
Cold application reduces swelling and minimizes secondary bleeding, but it does not provide immediate hemostasis.
It is supportive care rather than a primary intervention for active bleeding.
D. Apply a tourniquet below the wound:
A tourniquet should never be applied below the wound, as it would not effectively stop bleeding.
Tourniquets are used only for severe arterial bleeding, not for venous bleeding.
E. Apply a tourniquet above the wound:
A tourniquet is not recommended for venous bleeding as it completely stops blood flow to the limb, potentially causing ischemia.
Tourniquets are used only in life-threatening arterial hemorrhages when a pressure bandage is ineffective.
Conclusion:
Since the bleeding is steady and dark red, it indicates venous bleeding, which is best managed by applying a pressure bandage. Thus, option C is the correct answer.
36. A 6-year-old child complains of an unsteady gait, limb tremor, and fever. Accordi- ng to the patient’s medical history, the child has been ill with chickenpox for the past 4 days. Objectively, the child has nystagmus and dysarthria. What complication has developed in the child?
A. Meningitis
B. Brain abscess
C. Epilepsy
D. Encephalitis
E. Polyneuropathy
Correct Answer: D. Encephalitis
Explanation:
The child presents with neurological symptoms such as unsteady gait (ataxia), limb tremor, nystagmus, and dysarthria (difficulty speaking), along with fever. Given that the child had chickenpox (varicella) four days ago, the most likely complication is varicella encephalitis.
🔹 Varicella-associated encephalitis is a known neurological complication of chickenpox, typically occurring 3–7 days after the rash appears. It often presents with:
- Ataxia (unsteady gait)
- Tremors
- Nystagmus
- Dysarthria
- Fever
Why Other Options Are Incorrect:
A. Meningitis – Incorrect because:
- Meningitis typically presents with neck stiffness (meningeal signs), photophobia, and altered consciousness, which are not mentioned in this case.
- The child’s symptoms are more suggestive of cerebellar involvement rather than meningeal irritation.
B. Brain abscess – Incorrect because:
- A brain abscess usually presents with headache, focal neurological deficits, and signs of increased intracranial pressure (vomiting, papilledema).
- The child’s history of chickenpox makes encephalitis more likely.
C. Epilepsy – Incorrect because:
- Epilepsy is characterized by seizures, but there is no mention of seizures or loss of consciousness in this case.
E. Polyneuropathy – Incorrect because:
- Polyneuropathy typically presents with bilateral limb weakness, sensory loss, and absent reflexes.
- It does not explain ataxia and dysarthria, which are cerebellar rather than peripheral nerve issues.
Conclusion:
The child’s neurological symptoms occurring after a chickenpox infection strongly indicate varicella encephalitis, making encephalitis (option D) the most likely diagnosis.
- A 14-year-old patient complains of pain that occurs on an empty stomach and at night, nausea, and constipations observed over the last year. Endoscopy detects erosive bulbi- tis and increased secretory function of the stomach. H. pylori antigen test in feces is positive. What type of drug should be a component of the complex treatment in this case, as an antisecretory agent?
A. Bismuth preparation
B. Proton pump inhibitor
C. Histamine H₂ blocker
D. Selective muscarinic antagonist
E. Antacid
Correct Answer: B. Proton pump inhibitor
Explanation:
The 14-year-old patient presents with symptoms indicative of gastritis, likely due to Helicobacter pylori (H. pylori) infection, which is confirmed by the positive H. pylori antigen test. The patient has erosive bulbitis (inflammation of the duodenum) and increased gastric secretory function, leading to gastric irritation.
In the treatment of H. pylori-related gastritis and peptic ulcers, an antisecretory drug is required to reduce gastric acid secretion, promoting healing of the mucosa and enhancing the efficacy of antibiotics used to eradicate H. pylori.
🔹 Proton pump inhibitors (PPIs) are the most effective antisecretory agents, as they inhibit the proton pump (H⁺/K⁺ ATPase) in the parietal cells, significantly reducing gastric acid secretion and aiding in ulcer healing.
Why Other Options Are Incorrect:
A. Bismuth preparation – Incorrect because:
- Bismuth preparations (e.g., bismuth subsalicylate) are used as part of the combination therapy to treat H. pylori infection, but they are not potent antisecretory agents.
- They help to protect the gastric mucosa and have some antimicrobial activity, but they do not significantly reduce gastric acid production.
C. Histamine H₂ blocker – Incorrect because:
- While H₂ blockers (e.g., ranitidine, famotidine) reduce acid secretion, they are less effective than PPIs for severe conditions like H. pylori-related ulcer disease.
- PPIs provide stronger acid suppression, which is crucial for healing erosive lesions and ulcers.
D. Selective muscarinic antagonist – Incorrect because:
- Muscarinic antagonists (e.g., atropine) are not commonly used in the treatment of gastritis or H. pylori infection.
- They may reduce gastric acid secretion but have significant side effects and are less effective than PPIs.
E. Antacid – Incorrect because:
- Antacids (e.g., magnesium hydroxide, aluminum hydroxide) provide short-term relief of gastric irritation but do not adequately address the underlying H. pylori infection or the need for long-term acid suppression.
- They are not sufficient in treating gastritis or ulcers effectively.
Conclusion:
For this patient with erosive bulbitis and H. pylori infection, the most appropriate antisecretory agent is a proton pump inhibitor (PPI) (option B), which will provide effective acid suppression to promote healing and symptom relief.
38. A 5-year-old child complains of a cough that gradually intensifies. According to the patient’s medical history, the onset of the di- sease was 12 days ago. Objectively, the body temperature is 371°C, the skin is pale, the oropharyngeal mucosa is pale pink. There is a small ulcer on the frenulum of the tongue. Coughing fits occur up to 20 times per 24 hours and can be characterized by a series of cough impulses, followed by a wheezing inhalation. A coughing fit ends with the expectoration of vitreous sputum. Auscultation detects harsh respiration and dry (sometimes wet) crackles in the lungs. Chest X-ray shows a horizontal position of the ribs, increased transparency of the lungs, and an expanded pulmonary pattern in the roots of the lungs. Complete blood count: leukocytosis 23 109/L, eosinophils -1%, band neutrophils 3%, segmented neutrophils 23%, lymphocytes 70%, monocytes 3%, ESR-3 mm/hour. What is the most likely diagnosis in this case?
A. Laryngitis
B. Bronchitis
C. Pneumonia
D. Pertussis
E. Tuberculosis
Correct Answer: D. Pertussis
Explanation:
The child presents with severe cough fits that have progressively worsened over 12 days, with a characteristic pattern of a series of cough impulses followed by a wheezing inhalation. This is a hallmark of whooping cough or pertussis, a highly contagious bacterial infection caused by Bordetella pertussis.
Key features supporting the diagnosis of pertussis:
- Coughing fits: The hallmark of pertussis is a paroxysmal cough characterized by a series of coughs followed by a wheezing inspiratory “whoop”.
- Vitreous sputum: The child is expectorating glass-like (viscous) sputum, which is also consistent with pertussis.
- Increased lymphocytes (70%): The lymphocytic predominance in the complete blood count (CBC) is indicative of a viral or bacterial infection, and pertussis typically shows a marked increase in lymphocytes.
- Ulcer on the frenulum: A small ulcer on the frenulum of the tongue may be a sign of the associated mucosal irritation that can occur in pertussis.
- Radiographic findings: Chest X-ray showing an increased transparency of the lungs and an expanded pulmonary pattern with horizontal ribs are characteristic of atelectasis or air trapping seen in pertussis.
- Normal or mild fever: The child has a low-grade fever, which is commonly seen in pertussis but is not as pronounced as in more severe respiratory infections.
Given the duration of cough, the specific characteristics of the cough, and the immunologic response (lymphocytosis), the most likely diagnosis is pertussis.
Why Other Options Are Incorrect:
A. Laryngitis – Incorrect because:
- Laryngitis typically presents with a hoarse voice and a dry cough. It is generally associated with inflammation of the larynx rather than the coughing fits seen in pertussis.
- The characteristic “whoop” and coughing fits are not typical for laryngitis.
B. Bronchitis – Incorrect because:
- Bronchitis can cause a persistent cough, but it typically does not produce the paroxysmal cough or wheezing that is characteristic of pertussis.
- The increased lymphocytes seen in pertussis are not usually as pronounced in bronchitis.
C. Pneumonia – Incorrect because:
- Pneumonia usually presents with more severe systemic symptoms, such as higher fever and more prominent signs of respiratory distress (e.g., rapid breathing, significant respiratory failure).
- The cough in pneumonia is typically more productive and associated with purulent sputum, unlike the vitreous sputum seen in pertussis.
- The radiographic findings are not typical for pneumonia either, as pneumonia typically shows consolidation rather than the increased transparency and expanded lung pattern seen in this case.
E. Tuberculosis – Incorrect because:
- Tuberculosis (TB) typically presents with a chronic cough, night sweats, weight loss, and a history of exposure to TB.
- The acute presentation with paroxysmal cough, the absence of weight loss, and the normal ESR in this case make TB an unlikely diagnosis.
- Chest X-ray findings in TB usually show infiltrates or cavitation, which are not present here.
Conclusion:
The child’s paroxysmal cough, lymphocytosis, and characteristic radiographic findings strongly suggest pertussis (whooping cough). The treatment involves antibiotics (such as azithromycin) to reduce transmission and support the child’s recovery.
39. A 28-year-old patient complains of periodic palpitation attacks. Objectively, the following is observed: blood pressure 130/80 mm Hg, pulse 65/min, the heart sounds are unchanged. ECG shows shortening of the P-Q interval, an additional excitation wave appearing in the QRS complex, deformation of the QRS complex, and a negative P wave. What is the most likely diagnosis in this case?
A. Paroxysmal supraventricular tachycardia
B Ventricular preexcitation syndrome (WPW syndrome)
C. Paroxysmal ventricular tachycardia
D. Bundle branch block
E. Ventricular extrasystole
Correct Answer: B. Ventricular preexcitation syndrome (WPW syndrome)
Explanation:
The patient presents with periodic palpitation attacks. The ECG findings of shortening of the P-Q interval, an additional excitation wave in the QRS complex, deformation of the QRS complex, and a negative P wave are classic signs of Wolff-Parkinson-White (WPW) syndrome, a type of ventricular preexcitation syndrome.
Key points supporting WPW syndrome:
- Shortened P-Q interval: This suggests early activation of the ventricles due to an accessory pathway bypassing the normal AV node conduction.
- Additional excitation wave in the QRS complex: The presence of a delta wave, which is an initial slurring or “pre-excitation” in the QRS complex, is characteristic of WPW syndrome. This indicates the conduction of electrical impulses through an accessory pathway (bundle of Kent) that bypasses the AV node.
- Deformation of the QRS complex: The QRS complex in WPW syndrome is usually broader and deformed, showing abnormal conduction due to the accessory pathway.
- Negative P wave: A negative P wave can be indicative of retrograde conduction, where the impulse travels backward from the ventricles to the atria, which is common in WPW syndrome.
Why Other Options Are Incorrect:
A. Paroxysmal supraventricular tachycardia (PSVT) – Incorrect because:
- PSVT typically presents with a narrow QRS complex (less than 120 ms) and normal PR interval. The ECG findings of WPW syndrome (such as the delta wave and shortened P-Q interval) are distinct from those seen in PSVT.
- PSVT usually involves reentrant circuits involving the AV node, whereas WPW syndrome involves an accessory pathway.
C. Paroxysmal ventricular tachycardia – Incorrect because:
- Ventricular tachycardia (VT) typically presents with a wide QRS complex (greater than 120 ms) and often is associated with more severe symptoms, such as hypotension, dizziness, or syncope.
- The ECG findings described here (such as the delta wave and shortened PR interval) are not characteristic of VT.
D. Bundle branch block – Incorrect because:
- A bundle branch block (BBB) is characterized by a wide QRS complex (greater than 120 ms) due to delayed conduction in one of the bundle branches, leading to altered ventricular depolarization.
- The ECG findings described (such as the delta wave and shortened PR interval) are not typical of a bundle branch block.
E. Ventricular extrasystole – Incorrect because:
- Ventricular extrasystoles (premature ventricular contractions) present with early QRS complexes that are typically wide and bizarre in shape. They do not cause the characteristic QRS deformation seen in WPW syndrome, nor do they show the shortened PR interval or delta wave.
Conclusion:
The most likely diagnosis is Wolff-Parkinson-White (WPW) syndrome, a condition where an accessory pathway causes early ventricular activation and results in the characteristic ECG findings of shortened P-Q interval, delta wave, and deformed QRS complex. This is a form of ventricular preexcitation syndrome, and treatment may involve medications (such as antiarrhythmic agents) or procedures like radiofrequency catheter ablation to eliminate the accessory pathway.
40. A 12-year-old girl complains of general weakness, rapid fatigability, and intense periodical pain in her upper abdomen. The pain occurs immediately after eating or 15-20 minutes later. Periodically, she develops pain at night, sour cructation, and vomiting with just eaten food. According to the patient’s medical history, she has been suffering from chronic gastritis for the past 3 years. The patient’s father has a long history of peptic ulcer disease of the duodenum. Objectively, the patient’s skin is pale, the tongue has a white coating, the abdomen is soft and painful in the epigastrium during palpation. Mendel’s sign is positive. What is the most likely diagnosis in this case?
A. Chronic pancreatitis
B. Crohn’s disease
C. Functional dyspepsia
D. Peptic ulcer disease of the stomach
E. Chronic gastroduodenitis
Correct Answer: D. Peptic ulcer disease of the stomach
Explanation:
The patient presents with a history of chronic gastritis and a family history of peptic ulcer disease (PUD), which raises suspicion for an ulcer in the stomach or duodenum.
The symptoms that support this diagnosis include:
- Upper abdominal pain after eating, often characteristic of gastric ulcers.
- Sour regurgitation and vomiting after meals, which are signs of gastric distress and potential gastric outlet obstruction.
- Pale skin and white coating on the tongue, which may suggest a degree of anemia due to chronic blood loss or poor digestion in the stomach.
- The positive Mendel’s sign (pain in the epigastrium on palpation), which is a sign of gastric irritation or inflammation.
- A family history of duodenal ulcer in her father increases the likelihood of a genetic predisposition to peptic ulcer disease.
These findings are highly suggestive of peptic ulcer disease of the stomach, often linked to Helicobacter pylori infection or NSAID use, both of which could exacerbate the patient’s chronic gastritis.
Why Other Options Are Incorrect:
A. Chronic pancreatitis – Incorrect because:
- While chronic pancreatitis can cause upper abdominal pain, it typically presents with pain that radiates to the back and is often associated with steatorrhea, weight loss, and diarrhea, none of which are mentioned here.
- The patient’s symptoms are more consistent with gastric issues rather than pancreatic pathology.
B. Crohn’s disease – Incorrect because:
- Crohn’s disease usually affects the small intestine and/or colon, and is associated with symptoms such as chronic diarrhea, weight loss, and malabsorption, which are not present in this patient.
- Additionally, Crohn’s disease usually involves more diffuse, chronic, and recurrent gastrointestinal issues, which do not fit the episodic pain pattern described here.
C. Functional dyspepsia – Incorrect because:
- Functional dyspepsia involves discomfort in the upper abdomen without an identifiable underlying pathology like an ulcer, and may be associated with bloating, early satiety, or nausea, but pain triggered by eating and vomiting with food point more to a gastric ulcer.
- The positive Mendel’s sign further supports a diagnosis of a gastric ulcer or other organic pathology, rather than functional dyspepsia.
E. Chronic gastroduodenitis – Incorrect because:
- While gastroduodenitis involves inflammation of both the stomach and duodenum, the specific pain pattern occurring after meals, the family history of peptic ulcer, and the positive Mendel’s sign point more toward a gastric ulcer.
- Chronic gastroduodenitis typically causes more diffuse abdominal discomfort rather than localized pain.
Conclusion:
The most likely diagnosis in this patient is peptic ulcer disease of the stomach, which fits her history of chronic gastritis, upper abdominal pain after meals, and the family history of peptic ulcer disease. Treatment typically involves H. pylori eradication therapy (if applicable), proton pump inhibitors (PPIs), and lifestyle modifications.
41. A 54-year-old patient complains of heaviness in the right hypochondrium, yellowing of the skin and sclera, dark urine and colorless stools. According to the patient’s medical hi- story, the jaundice has been persisting for the last 3 weeks and before that the patient was noting general weakness. Objectively, the patient’s condition is satisfactory, the skin and sclerae are icteric. The abdomen is soft, an enlarged painless gallbladder can be palpated in the right hypochondrium. There are no signs of peritoneal irritation. What is the most likely diagnosis in this case?
A. Acute cholecystitis
B. Hemolytic jaundice
C. Choledocholithiasis
D. Acute hepatitis
E. Pancreatic head cancer
Correct Answer: E. Pancreatic head cancer
Explanation:
This patient presents with painless progressive jaundice, an enlarged, palpable gallbladder (Courvoisier’s sign), dark urine, and acholic stools, which strongly suggest obstructive jaundice. The most common cause of painless obstructive jaundice with a palpable gallbladder is pancreatic head cancer, which compresses the common bile duct (CBD), leading to bile flow obstruction.
Additional clues include the chronic progression over weeks, absence of acute symptoms (such as fever or severe pain), and the lack of peritoneal signs, which help differentiate this condition from other causes of jaundice.
Why Other Options Are Incorrect:
- A. Acute cholecystitis – Incorrect.
- Acute cholecystitis typically presents with fever, right upper quadrant pain (Murphy’s sign), nausea, and vomiting.
- The gallbladder in acute cholecystitis is usually tender, whereas this patient’s gallbladder is painless.
- B. Hemolytic jaundice – Incorrect.
- Hemolytic jaundice results from excessive breakdown of red blood cells, leading to unconjugated hyperbilirubinemia.
- It does not cause dark urine (since unconjugated bilirubin is not water-soluble and cannot be excreted in urine) or acholic stools (since bile production is not obstructed).
- C. Choledocholithiasis – Incorrect.
- While choledocholithiasis (common bile duct stones) can cause jaundice, dark urine, and acholic stools, it is typically associated with colicky right upper quadrant pain, which is absent here.
- If prolonged, it can lead to cholangitis (fever, jaundice, right upper quadrant pain – Charcot’s triad), which this patient does not have.
- D. Acute hepatitis – Incorrect.
- Acute hepatitis presents with jaundice but is often associated with malaise, nausea, vomiting, abdominal discomfort, and elevated liver enzymes.
- It does not cause a palpable gallbladder or acholic stools, as bile flow is not obstructed.
Conclusion:
The presence of painless jaundice, Courvoisier’s sign (enlarged, non-tender gallbladder), and progressive symptoms over weeks strongly suggest pancreatic head cancer as the most likely diagnosis.
42. A 42-year-old woman complains of general weakness and producing bloody discharge at the end of an act of defecation. Examination with a rectal speculum detects soft, painless, dark cherry-red protrusions with an eroded surface in the anal canal. What is the most likely diagnosis in this case?
A. Crohn’s disease
B-Internal hemorrhoids
C. Rectal tumor
D. Chronic paraproctitis
E. Rectal polyp
Correct Answer: B. Internal hemorrhoids
Explanation:
The patient’s symptoms of general weakness, bloody discharge at the end of defecation, and the presence of soft, painless, dark cherry-red protrusions with an eroded surface in the anal canal strongly suggest internal hemorrhoids.
Internal hemorrhoids typically present with:
- Painless bleeding during or after defecation, often described as bright red blood.
- Protruding, swollen veins in the anal canal, which may be seen as soft, dark red or cherry-colored masses.
- These masses can become eroded or ulcerated, especially if there is repeated trauma from passing stools.
Why Other Options Are Incorrect:
A. Crohn’s disease – Incorrect because:
- Crohn’s disease can affect any part of the gastrointestinal tract, including the rectum, and may present with rectal bleeding and ulcers. However, the protrusions in this case are painless, and Crohn’s disease typically causes other symptoms such as diarrhea, abdominal pain, and systemic signs of inflammation (e.g., fever, weight loss). Internal hemorrhoids are much more likely given the description of the protrusions.
C. Rectal tumor – Incorrect because:
- A rectal tumor may cause rectal bleeding, but typically there would be painless rectal bleeding at the beginning of defecation (due to the tumor’s location). The protrusions in this case appear to be painless, and there is no mention of a mass or hard lesion, which is more characteristic of a tumor. Moreover, rectal tumors tend to be associated with more serious systemic symptoms like weight loss or altered bowel habits, which are absent here.
D. Chronic paraproctitis – Incorrect because:
- Chronic paraproctitis (or anal fistulas) presents with painful abscesses or fistulas in the perianal area. The soft, painless, cherry-red protrusions in this patient do not fit with paraproctitis, which would likely cause pain, discharge, and possible swelling around the anus.
E. Rectal polyp – Incorrect because:
- Rectal polyps are usually asymptomatic or cause occult bleeding (not visible to the eye). The painless, cherry-red protrusions described here are more likely to be hemorrhoids. Polyps may not present with the same acute symptoms and the described appearance.
Conclusion:
The most likely diagnosis is internal hemorrhoids, which are characterized by painless, dark protrusions in the anal canal, often associated with rectal bleeding. Internal hemorrhoids occur when veins in the rectum become swollen and dilated, often due to increased pressure from straining or constipation. Treatment may include lifestyle changes, topical treatments, or in severe cases, surgical intervention.
43. A 42-year-old patient has been hospitalized in a severe condition with seizures. Objectively, he has a sardonic smile and nuchal rigidity, Stimulation provokes clonic seizures in the limbs. Pulse 140/min, respiratory rate 30/min. According to the patient’s wife, three weeks ago he injured his leg with a rusty nail. What is the most likely diagnosis in this case?
A. Tetanus
B. Rabies
C. Meningitis
D. Erysipelas
E. Tick-borne encephalitis
Correct Answer: A. Tetanus
Explanation:
The patient’s symptoms of seizures, sardonic smile, nuchal rigidity, and clonic seizures in the limbs are highly suggestive of tetanus. This is particularly likely given the history of an injury involving a rusty nail three weeks ago, as tetanus is commonly caused by a Clostridium tetani infection following wounds or trauma.
Tetanus symptoms:
- Sardonic smile (risus sardonicus): A characteristic sign of tetanus, where facial muscles contract, giving the patient an abnormal, forced smile.
- Nuchal rigidity: Stiffness in the neck muscles, commonly seen in tetanus.
- Clonic seizures: Involuntary muscle contractions and spasms, often triggered by sensory stimuli or muscle stretch.
- Increased pulse and respiratory rate: These can be due to the autonomic dysfunction associated with tetanus.
Why Other Options Are Incorrect:
B. Rabies – Incorrect because:
- Rabies typically presents with symptoms like agitation, hydrophobia (fear of water), aerophobia (fear of air drafts), and paralysis in the later stages. The sardonic smile is not characteristic of rabies.
- Rabies also has a different course, often progressing to coma and respiratory failure after symptoms like hydrophobia and hypersalivation.
C. Meningitis – Incorrect because:
- While meningitis can present with nuchal rigidity and seizures, it would typically not cause a sardonic smile or clonic seizures induced by stimulation. Additionally, meningitis often has a more rapid onset of symptoms and is associated with fever, headache, and photophobia, which are not mentioned here.
D. Erysipelas – Incorrect because:
- Erysipelas is a superficial bacterial skin infection, characterized by red, swollen, and raised lesions, often on the face or legs. It would not explain the neurological signs such as seizures, sardonic smile, or nuchal rigidity. The wound described here is more consistent with tetanus than erysipelas.
E. Tick-borne encephalitis – Incorrect because:
- Tick-borne encephalitis typically presents with symptoms of fever, headache, and neurological symptoms, but it does not usually present with sardonic smile or clonic seizures as seen in tetanus. Additionally, the patient’s history does not mention a tick bite.
Conclusion:
Given the history of injury with a rusty nail, the neurological signs (e.g., sardonic smile, clonic seizures), and autonomic signs (e.g., tachycardia and tachypnea), the most likely diagnosis is tetanus. Immediate antitoxin therapy, muscle relaxants, and supportive care (including mechanical ventilation) are critical for managing tetanus.
44. A 38-year-old man complains of cough with purulent sputum (up to 60-80 mL per 24 hours) and fever of 39°C. He associates his condition with overexposure to cold. Objecti- vely, his pulse is 96/min, rhythmic. Blood pressure 110/60 mm Hg. Examination reveals that the right side of the chest is lagging behind in the process of breathing. Respiratory rate 30/min. Percussion detects local dullness of the sound near the angle of the scapula. Auscultation detects heterogeneous wet crackles and amphoric breathing. What is the most likely diagnosis in this case?
A. Acute lung abscess
B. Pneumonia
C. Bronchial asthma
D. Acute bronchitis
E. Pleural empyema
Correct Answer: A. Acute lung absces
Explanation:
The patient presents with a productive cough with purulent sputum, fever, and localized dullness on percussion. The clinical findings suggest the presence of a lung abscess:
- Purulent sputum (60-80 mL/day) is highly suggestive of a lung abscess, as abscesses often lead to the production of foul-smelling, purulent sputum.
- Fever (39°C) is a common sign of infection, and an abscess is typically associated with localized infection in the lung tissue.
- Lagging chest movement on the right side during respiration indicates a possible pleural effusion or complicated pneumonia, which is common in lung abscesses when infection spreads to the pleura.
- Dullness on percussion near the angle of the scapula points to consolidation or fluid accumulation (suggesting the formation of an abscess or effusion).
- Heterogeneous wet crackles and amphoric breathing (a characteristic finding in abscesses) further support the diagnosis of a lung abscess.
Why Other Options Are Incorrect:
B. Pneumonia – Incorrect because:
- Pneumonia can present with fever, cough, and purulent sputum, but it does not typically lead to localized dullness on percussion or amphoric breath sounds, which are more characteristic of a lung abscess.
- Pneumonia is usually more diffuse on physical exam compared to the localized findings seen in this case.
C. Bronchial asthma – Incorrect because:
- Bronchial asthma generally presents with wheezing, dyspnea, and reversible airway obstruction, not with a productive cough with purulent sputum or localized dullness on percussion.
- Asthma exacerbations do not present with amphoric breathing.
D. Acute bronchitis – Incorrect because:
- Acute bronchitis presents with cough (often non-productive or with clear sputum), wheezing, and dyspnea, but purulent sputum and localized dullness on percussion are not typically seen.
- Acute bronchitis is more generalized and would not cause localized dullness or amphoric breathing.
E. Pleural empyema – Incorrect because:
- While pleural empyema (infected pleural effusion) can cause dullness on percussion and fever, the amphoric breathing and purulent sputum are more suggestive of a lung abscess.
- In pleural empyema, friction rub or dullness would be more commonly associated with pleural involvement rather than the localized findings seen in a lung abscess.
Conclusion:
The most likely diagnosis is acute lung abscess, given the purulent sputum, fever, localized dullness, and amphoric breath sounds, which are characteristic findings of a lung abscess. Treatment typically includes antibiotics, and in some cases, drainage may be required.
45. A 20-year-old woman complains of fever of 38°C and productive cough with expectoration of sputum. According to the patient’s medical history, the disease onset was acute and occurred 5 days ago after recovery from a case of an acute respiratory viral infection. Objectively, respiratory rate 26/min. Percussion produces a dull sound in the lower segments of the lungs on the right. Auscultation detects weakened respiration in this area, with wet fi- ne vesicular crackles. Heart rate 110/min. What is the most likely diagnosis in this case?
A. Bronchial asthma
B. Pyopneumothorax
C Pneumonia
D. Acute bronchitis
E. Tuberculosis
Correct Answer: C. Pneumonia
Explanation:
The patient’s presentation, which includes fever (38°C), productive cough with sputum expectoration, acute onset of symptoms after a recent respiratory viral infection, and localized dullness on percussion, strongly suggests pneumonia:
- Acute onset of symptoms after an upper respiratory infection (such as a viral infection) is common in cases of bacterial pneumonia. The viral infection may predispose the lungs to a secondary bacterial infection.
- Fever (38°C) and productive cough are classic symptoms of pneumonia.
- The dullness on percussion in the lower segments of the right lung points to consolidation, which is a key sign of pneumonia, typically due to bacterial infection.
- Wet fine crackles on auscultation also suggest consolidation, a hallmark of pneumonia.
- The increased heart rate (tachycardia, 110/min) is a common response to infection, especially in the setting of pneumonia.
Why Other Options Are Incorrect:
A. Bronchial asthma – Incorrect because:
- Asthma typically presents with wheezing, shortness of breath, and reversible airflow obstruction, not with productive cough and localized dullness on percussion.
- The wet fine crackles and dullness on percussion are not characteristic of asthma.
B. Pyopneumothorax – Incorrect because:
- Pyopneumothorax (infection associated with a pneumothorax) typically presents with pleuritic chest pain, dyspnea, and unilateral breath sounds, often with decreased lung volume or absent breath sounds on the affected side.
- The dullness and crackles on auscultation do not strongly support this diagnosis in the absence of signs of pneumothorax (such as hyper-resonance or absent breath sounds).
D. Acute bronchitis – Incorrect because:
- Acute bronchitis often presents with a cough, but it is typically non-productive or with clear sputum, and localized dullness on percussion and crackles are not commonly seen in acute bronchitis.
- Bronchitis does not cause the localized consolidation seen in pneumonia.
E. Tuberculosis – Incorrect because:
- Tuberculosis usually presents with chronic cough, hemoptysis, night sweats, and weight loss, rather than the acute onset and localized findings seen in this case.
- Tuberculosis is more often associated with upper lobe consolidation or cavitation, rather than the lower lung dullness and crackles seen here.
Conclusion:
The most likely diagnosis is pneumonia, given the acute onset, fever, productive cough, and findings of dullness on percussion and crackles on auscultation, which are consistent with consolidation in the lung. Further investigation, such as a chest X-ray, would be helpful for confirmation and to assess the extent of the pneumonia. Treatment typically involves antibiotics based on the likely bacterial pathogen.
46. Workers at the mining sites are exposed to mineral dust in the form of disintegration- produced aerosols. Chemical and physical properties of dust were studied to measure the risk of workers developing occupational lung diseases caused by dust. What characteristic of dust generally determines how deep into the respiratory tract can it reach?
A. Solubility
B. Dispersity
C. Silicon dioxide levels
D. Electric charge
E. Shape of dust particles
Correct Answer: B. Dispersity
Explanation:
Dispersity refers to the size distribution of the dust particles. The size of dust particles is the primary factor determining how deep they can penetrate into the respiratory tract. Smaller particles (typically less than 10 micrometers in diameter, known as PM10) can reach the lower airways and even the alveoli, while larger particles are usually trapped in the upper respiratory tract or the nasal passages.
- Fine particles (PM2.5), less than 2.5 micrometers, are even more likely to reach the deepest parts of the lungs and can cause more significant health problems over time, including pneumoconiosis (lung disease caused by inhalation of dust).
Why Other Options Are Incorrect:
A. Solubility – While solubility can influence the toxicity of the dust (whether it can cause chemical reactions once inside the lungs), it does not primarily determine how deep the particles can reach. The size and dispersity of the particles are the primary factors for lung penetration.
C. Silicon dioxide levels – High levels of silicon dioxide (such as in silica dust) can increase the risk of silicosis, but silicon content itself does not determine the depth of penetration. It’s the size of the particles that determines where they settle in the respiratory tract.
D. Electric charge – The electric charge of dust particles may affect how they interact with surfaces or each other, but it has little impact on how deeply they can penetrate into the respiratory system compared to particle size.
E. Shape of dust particles – The shape of dust particles can affect how they interact with the respiratory system and might influence how easily they are cleared by the body, but particle size (dispersity) is the primary determinant of how deep the particles can reach into the lungs.
Conclusion:
Dispersity (the size of dust particles) is the primary characteristic that determines how deep the dust can penetrate into the respiratory tract, with smaller particles able to reach the lower lungs and alveoli, posing a greater health risk.
47. A 54-year-old man has been hospitalized with complaints of general weakness, fever of 38.6°C, shortness of breath during significant physical exertion, and frequent nosebleeds. He works in production of plastics, where he comes into contact with aromatic compounds. Objectively, his skin is pale and dry. Complete blood count: erythrocytes 2.1 1012/L, hemoglobin 90 g/L, leukocytes 2.2-109/L, eosinophils 1%, band neutrophils 1%, segmented neutrophils 75%, lymphocytes 20%, myelocytes 3%, platelets 30109/L, ESR32 mm/hours. What is the most likely diagnosis in this case?
A. Chronic tetraethyllead intoxication
B. Chronic lead intoxication
C. Chronic aniline intoxication
D. Chronic nitrobenzene intoxication
E. Chronic benzene intoxication
Correct Answer: E. Chronic benzene intoxication
Explanation:
The patient’s occupation in the plastic production industry and symptoms suggest chronic exposure to benzene, a known occupational toxin. Chronic benzene intoxication can lead to:
- Bone marrow suppression, which may explain the patient’s low erythrocyte count (2.1 x 10¹²/L), low hemoglobin (90 g/L), and low leukocytes (2.2 x 10⁹/L).
- Anemia and leukopenia are characteristic findings in benzene poisoning due to its toxic effects on the bone marrow, which impairs the production of red and white blood cells.
- The patient’s shortness of breath, general weakness, fever, and frequent nosebleeds could be related to anemia (due to low hemoglobin levels) and thrombocytopenia, as well as immune suppression.
- Elevated ESR (32 mm/h) could suggest an inflammatory process, which can also be associated with bone marrow dysfunction.
Why Other Options Are Incorrect:
A. Chronic tetraethyllead intoxication – While lead poisoning can cause anemia, it typically presents with neuropsychiatric symptoms (e.g., irritability, headache), abdominal pain, and memory problems, which are not described in this patient. The history of exposure to aromatic compounds in plastic production makes benzene a more likely cause.
B. Chronic lead intoxication – Lead poisoning can lead to anemia and neurological symptoms, but this patient’s presentation is more consistent with benzene toxicity, especially considering the specific workplace exposure. Lead also does not commonly cause the leukopenia and myelocytes (immature white blood cells) seen here.
C. Chronic aniline intoxication – Chronic exposure to aniline can lead to methemoglobinemia and symptoms like cyanosis, but it is less commonly associated with bone marrow suppression and the hematologic findings seen here, especially the low white blood cell count.
D. Chronic nitrobenzene intoxication – Nitrobenzene poisoning can cause symptoms like headache, nausea, and cyanosis due to methemoglobinemia, but it is not typically associated with the severe bone marrow suppression seen in this case.
Conclusion:
The most likely diagnosis is chronic benzene intoxication, given the patient’s occupational exposure, hematologic abnormalities, and systemic symptoms such as anemia and leukopenia, which are consistent with the toxic effects of benzene on the bone marrow.
48 On the second day after the childbirth, the 28-year-old postparturient woman with severe preeclampsia against the background of intense headache and increased blood pressure of 180/110 mm Hg developed a seizure attack, after which she remained in the state of increased seizure readiness. What medicine should be prescribed in this case to prevent convulsive status in the patient?
A-Phenazepam
B. Haloperidol
C. Magnesium sulfate
D. Chlorpromazine
E. Promedol (Trimeperidine)
Correct Answer: C. Magnesium sulfate
Explanation:
This patient is experiencing eclampsia, a severe complication of preeclampsia characterized by seizures in a woman with hypertension and proteinuria. Magnesium sulfate is the drug of choice for preventing and treating eclamptic seizures because it stabilizes neuronal membranes, reduces neuromuscular excitability, and acts as a central nervous system depressant.
Why Other Options Are Incorrect:
- A. Phenazepam (Benzodiazepine) – Incorrect.
- Although benzodiazepines like diazepam or lorazepam can be used for seizure control in other conditions, they are not first-line for eclampsia.
- Magnesium sulfate is preferred because it reduces seizure recurrence and lowers blood pressure indirectly by relaxing vascular smooth muscle.
- B. Haloperidol (Antipsychotic) – Incorrect.
- Haloperidol is used for psychotic disorders and agitation, not for seizure prevention in eclampsia.
- It has no role in managing hypertensive emergencies or seizure control in this setting.
- D. Chlorpromazine (Antipsychotic and Sedative) – Incorrect.
- Chlorpromazine is primarily used for psychiatric conditions and sedation.
- It does not prevent seizures and has hypotensive effects that may worsen maternal hemodynamics unpredictably.
- E. Promedol (Trimeperidine, Opioid Analgesic) – Incorrect.
- Opioids are used for pain relief but do not prevent or treat seizures.
- Additionally, opioids can cause respiratory depression, which is dangerous in eclamptic patients.
Conclusion:
For eclamptic seizures, magnesium sulfate is the treatment of choice due to its superior efficacy in preventing convulsive status and reducing the risk of seizure recurrence..
49. A 26-year-old man complains of a rash on his face, itching, feeling of heat, and pain. According to the patient’s medical history, this condition has been observed over the last two years. Objectively, there are infiltrated bri- ght red patches on the skin of his face in the area of the chin. The patches are covered in pustules, scabs, and numerous erosions and have papulo-pustular elements located on their periphery. What is the most likely di- agnosis in this case?
A. Scabies
B. Eczema
C. Contact dermatitis
D-Sycosis
E. Acne vulgaris
Correct Answer: D. Sycosis
Explanation:
Sycosis is a chronic staphylococcal infection of the hair follicles, commonly affecting the beard area (sycosis barbae). It is characterized by:
- Bright red, inflamed patches with pustules and crusting
- Chronic course with remissions and exacerbations
- Itching, burning, and tenderness
- Papulopustular lesions at the periphery of the affected area
This patient’s two-year history of recurrent inflammatory pustular eruptions localized to the chin area strongly suggests sycosis rather than other differential diagnoses.
Why Other Options Are Incorrect:
- A. Scabies – Incorrect.
- Scabies presents with intense nocturnal itching and characteristic burrows, typically in the interdigital spaces, wrists, axillae, and genital area.
- It does not present with localized pustules and scabs on the face.
- B. Eczema – Incorrect.
- Eczema (atopic dermatitis) presents with pruritic, dry, scaly skin and can involve vesicles or lichenification, but it lacks pustules and erosions.
- Chronic eczema may cause lichenification, but it is usually not localized specifically to the chin.
- C. Contact dermatitis – Incorrect.
- Contact dermatitis typically follows exposure to an irritant or allergen and presents with erythema, vesicles, and oozing lesions rather than pustules and chronic scabbing.
- It resolves with avoidance of the triggering substance, unlike this chronic, persistent case.
- E. Acne vulgaris – Incorrect.
- Acne presents with comedones (blackheads and whiteheads), papules, pustules, and nodules, primarily on the forehead, cheeks, nose, and back rather than being localized to the beard area.
- It is not associated with extensive erosions and crusting seen in sycosis.
Conclusion:
Given the localized pustular inflammation in the beard area with a chronic course, the most likely diagnosis is sycosis barbae, a superficial bacterial folliculitis caused by Staphylococcus aureus.
50 A 42-year-old patient has been hospitalized with complaints of constant pain in the lumbar spine. The pain intensifies during movements. According to the patient’s history, he fell from the height of the second floor during construction work. Objectively, smoothing of the lumbar lordosis is observed. During palpation of the spinous processes, the pain is most intense at the level of [L₁ vertebra. “Stuck heel” sign is positive (inability to perform the straight leg raise test). What is the most likely diagnosis in this case?
A L₁ vertebral body compression fracture
B. Contusion of the soft tissues in the lumbar region
C. Contusion of the lumbar spine
D Femoral neck fracture
E. Exacerbation of osteochondrosis of the lumbar spine
Correct Answer: A. L₁ vertebral body compression fracture
Explanation:
The patient’s history of falling from a height, severe localized pain in the lumbar spine, and “stuck heel” sign positivity strongly suggest a vertebral compression fracture, specifically at the L₁ vertebra.
- Compression fractures often occur in the thoracolumbar junction (T12–L2) due to high-energy trauma (e.g., falls).
- Smoothing of lumbar lordosis suggests a protective muscle spasm due to the fracture.
- The “stuck heel” sign (inability to raise the straight leg due to pain) indicates spinal pathology.
- Intense pain on palpation of the L₁ spinous process further supports the presence of a fracture.
Why Other Options Are Incorrect:
- B. Contusion of the soft tissues in the lumbar region – Incorrect.
- Soft tissue contusions cause localized swelling and bruising but do not cause severe movement-restricting pain or positive “stuck heel” sign.
- C. Contusion of the lumbar spine – Incorrect.
- While a spinal contusion may cause pain and swelling, it does not typically cause focal tenderness at a specific vertebra or structural deformity (like compression fractures).
- Contusions also lack neurological signs and do not cause persistent severe pain with movement.
- D. Femoral neck fracture – Incorrect.
- Femoral neck fractures present with hip pain, external rotation, and shortening of the affected leg, not lumbar pain.
- A positive “stuck heel” sign is not characteristic of femoral neck fractures.
- E. Exacerbation of osteochondrosis of the lumbar spine – Incorrect.
- Lumbar osteochondrosis (degenerative disc disease) causes chronic, intermittent back pain with periods of relief and exacerbation.
- It does not present with acute, trauma-related pain or focal vertebral tenderness.
Conclusion:
The history of trauma (fall from height), localized lumbar pain, “stuck heel” sign positivity, and focal vertebral tenderness make L₁ vertebral body compression fracture the most likely diagnosis..